Sie sind auf Seite 1von 49

CRIMINAL PROCEDURE KING S07 Prosecutor GJ Grand Jury GP Guilty plea BRD-Beyond Reason.

ason. Doubt USSG PCR state post-conviction relief FRCrP BRA Bail Reform Act I. INTRODUCTION PROCESS OVERVIEW
Arrest w/o Warrant Booking Detention Screening Charge Screening Complaint First Appearance Preliminary Hearing Pleas Motions Trial Sentencing Appeal [State post conviction review] Federal Habeas Corpus Arrest w/ warrant Booking Charge Screening Complaint First Appearance Preliminary Hearing Pleas Motions Trial Sentencing Appeal [State post conviction review] Federal Habeas Corpus Arrest after GJ invest. / indict.

PC Probable Cause DJ double jeopardy

PHrg Preliminary Hearing DPen Death penalty

FHC- fed. Habeas corpus DP due process STA Speedy Trial Act

Pleas Motions Trial Sentencing Appeal [State post conviction review] Federal Habeas Corpus

[Magistrate Level] Booking: charge screening by police o Decision whether accused stays or goes based on schedules Detention Screening: required when arrest made without warrant and keeping accused >48hr o Gerstein REVIEW: determination of PC for offense charged in complaint Ex parte; Magistrate judge and police officer Charge Screening: reviews police investigation so far; can conduct post-arrest investigation Complaint: usually filed by victim or police officer later becomes Information or Indictment] o Usually filed by victim or police First Appearance: Magistrate sets bail and PHrg date; arranges attorney if necessary and officers present Preliminary Hearing: Last appearance in Magistrate and First critical adversarial step o reestablishes PC - s attorney can cross o Magistrate binds case over to felony court with an information now officially charged with felony o ALTERNATIVE: Grand Jury (GJ) essentially does same thing as Magistrate re-screens for PC *18 states have Constitutional guarantee for GJ review of an information BUT many waive this right so they dont have to wait in jail for GJ review Funnel System: At each stage, few and fewer s remain

SOURCES OF REGULATION Federal Constitution o Art III: All criminal trials shall be by jury but now viewed as waivable o Bill of Rights: Selective incorporation: only impose fundamental BoR Rights on states via 14A DP Duncan v. La. procedural right is fundamental if it is necessary in context of criminal processes maintained by US 4A: free form unreasonable search/seizure exclusionary rule 5A-DP: No DJ privilege against self-incrimination o *right to GJ indictment NOT incorporated
1

6A: Right to speedy trial public trial impartial jury jury trial informed of nature of charge confrontation compulsory process counsel in capital, felon, & jail term cases 8A: No cruel or unusual punishment o *no excessive bail or fines NOT specifically incorporated reverse incorporation: 14A EP applicable to federal s DP in fed court is 5A; state court 14A **most of s rights are WAIVABLE State Constitution Statutes for each Jx. Court rules FRCrP and state rules

SYSTEMIC CONCERNS - Systems structure dominated by o Crime rates o Definitions of crime o Funding decisions limits on s funding; increases in costs of prosecuting Legislatures have expanded s resources but -counsel spending has decreased o *3 factors interrelated: Eg. increased prosecuting costs broader definition of crime more GPs lower costs of prosecution - Constitutional focus of criminal law as opposed to substantive focus on underlying conduct o Often cheaper for to raise Constitutional procedural claim than to litigate on the merits and facts

II. DECISION TO PROSECUTE OVERVIEW Sources: o Constitution EP and DP o Court rules, statutes o Guidelines: ABA standards or internal office guidelines *carry No force of law Vertical Assignment System: one or several s get all cases docketed for particular judge or courtroom o Same on case through whole process Horizontal Assignment System: certain prosecutor assigned to handle case at various steps PROSECUTORS DISCRETION has near limitless discretion in decide whether to charge o FACTORS: must relate to public interest in or in not prosecuting Accused-related: *s reasonable doubt that accused is guilty Disproportionate punishment to particular accused and offense o Prior Record o Any personalized factors o Accused benefit better from alternative treatment Cooperation of accused in apprehension of others Victim-related: Extent of harm or injury caused Vs culpability mutual combat/domestic squabble Vs willingness to testify Possible improper motives for complaining Victim and Witness Protection Act of 1982 (VPA) Resource-related: Strength of admissible evidence Cost of prosecution Likelihood of jury nullification Likelihood of Plea Alternative remedies to criminal prosecution mediation, civil action, restitution Political & community pressures officials are elected and expected to do what citizens want Whether prosecution does more harm e.g. domestic disturbance, 1st time shoplifter Availability and likelihood of prosecution in another Jx o IMPROPER factors: Accused is friend or relative Race, gender, etc. has discretion in deciding which charge to bring o Constitutional for to select offense with higher mandatory sentence when selecting between two offenses covering identical conduct (Batchelder) RATIONALE: Judge ultimately decides the penalty and can determine relative culpability among offenders LIMITS ON DECISION NOT TO PROSECUTE Political System: Direct accountability to voting public CAN be valid limit o has duty to serve community interests Special Prosecutor: can be appointed by state/federal AG when local refuses to charge (Pataki) Substantive Law reform: if criminal offense on books just to threaten, then it could/should expire o Desuetude: theory that if criminal statue remains un-enforced long enough then Constitution forbids its enforcement
3

Grand Jury: CAN bring charges with out s consent o Option in many states but NOT in federal system Victim: NO prosecutorial power and can only bring civil action o Right to confer with , not prosecutor or veto only remedy is political process o RATIONALE: need administrative screening; prevent angry and frivolous claims Court can NOT force prosecutor to bring charges (Attica) o RATIONALE: Violates SoP: Judiciary cant interfere with Executive branch Also impractical: no standards by which judges could supervise s decisions Legislation: NO legislative or administrative requirements on when to charge o RATIONALE: Impractical: no manageable standards that could properly accommodate balancing all relevant factors . Beyond judicial capacity to supervise and insure follows standards

LIMITS ON DECISION TO PROSECUTE can NOT charge unless supported by Probable Cause (PC) that accused committed offense SELECTIVE PROSECUTION (EP violation) arbitrary selection o CLAIM: s right to equal protection violated when brings charge because: (1) or V is in a protected class OR (2) exercised a fundamental right o RULE: must show (1) discriminatory effect AND (2) discriminatory purpose (Armstrong) (1): prove by showing similarly situated [group] is NOT prosecuted (Aguilar) must show that its the same or s office that intends to prosecute selectively o general statistical analysis or evidence is insufficient (McCleskey) (2): must prove legitimate non-discriminatory purpose ***Very difficult to prove BUT if met, rebuttable presumption and burden shift to to show legitimate non-discriminatory purpose Discovery: is entitled to discovery of s records if he can show at least some evidence of both elements (1) and (2) (Armstrong) o RATIONALE: high level of deference to dont want to chill decision making or affect ongoing investigations need to keep plea bargain leverage dont divert s time and resources by forcing him to constantly defend these potentially frivolous o REMEDY is dismissal No civil remedies: s generally immune and no 1983 claim for selective or vindictive prosecution if has PC
o Armstrong (1996) attempts to get discovery for selective prosecution claim with evidence affidavit that 24/24 cases closed by this office had black s court held NOT enough of a colorable basis need to show some intent of discriminatory effect. had many valid reasons for selecting charge Aguilar (9th cir 1989) s argued group was 1A rights exercisers but court held more similar to alien smuggling conspiracies generally doesnt prosecute organized alien smugglersexclusive focus on employers status and 1A tries to get away from fact that their group is prosecuted s dont content McCleskey (1987) claims sought DPen b/c he was black and victim was white BUT failed to provide any evidence that, in his case, had discriminatory purpose General statistical study of DPen and race relations insufficient evidence

VINDICTIVE PROSECUTION (DP violation) improper motives o CLAIM: charges brought against because he exercised a procedural right (e.g. requested jury trial, retrial, etc) o RULE: (1) Presume vindictiveness where has interest in discouraging s exercise of a right OR (2) can show actual vindictiveness eitherway (3) can rebut with valid non-vindictive purpose (1) appeals, jury trial, de novo trial, situation where would spend more resources and time and has something he stands to lose (a conviction already-won) (Blackledge) NO presumption for threatening/increasing charge during plea negotiations NO presumption for exercise of pre-trial right
4

o o

(2) can must show actual retaliatory motive R2: after presumption or actual vindictiveness showing can rebut by showing objective evidence of legitimate non-vindictive purpose REMEDY is dismissal RATIONALE: would violate DP clause process not due if punished for electing to exercise a Constitutionally granted rights No presumption for pre-trial setting s still need full discretion, dont want the initial charging decision to be frozen in case circumstances/evidence changes
Pearce (1969) received higher sentence when re-sentenced after successful appeal H: presumption of vindictiveness unless judge gives affirmative reasons based on objective information concerning s identifiable conduct at time of original sentencing Blackledge (1974) asserts statutory right to de novo trial in higher court after magistrate trial; raises charges and secures felony indictment H: presumption applies; has interest in discouraging from exercising Bordenkircher (1978) admits vindictiveness when he told he would bring habitual criminal charge if didnt accept plea NO presumption of vindictiveness plea negotiation context give and take nature of plea negotiations, knew his choices *opposite holding could cripple plea negotiation system; plus, opposite holding wouldnt change anything s would just charge higher and drop them in context of pleas Goodwin (1982) in magistrate pre-trial and requests jury trial in district court; increases to felony charges NOT like Blackledge where had conviction to lose; here, just pre-trial, need actual vindiction

III. PRETRIAL DETENTION OVERVIEW Balance risks of release o ** Flight: main concern is wont show up to court once released to avoid retribution o Harm to others: concern that might cause danger to others if he returns to community with risks of detention o If acquitted will have served unnecessary time o Pretrial detention period may be longer than actual sentence o Hampers ability to prepare defense Limits ability to talk to counsel, collect witness Cant earn money to pay for attorney Stigma associated with being locked up o Collateral consequences: cant support family, etc. o Risk innocent s who plead guilty solely to avoid pretrial detention Important step in process: o Both sides of argument present compelling concerns o Impact n other steps of process STA and sentencing process RELEASE PROCEDURES Non-detention forms of release o Police decisions: Field release never bring in Stationhouse release released at booking stage, generally just pays cash for release o Magistrate decision at first appearance: Personal recognizance (PR): most lenient, only a promise to come back for next hearing Supervised release: non-secured conditions No collateral just flexible and customized conditions: o Phone in, ankle bracelet, live at home, stay employed and in town, drug test o Probation officer *Unsecured bond: most common form No collateral just conditions and a promise to pay if you fail to appear Other conditions set similar to supervised release Surety bond: bail Full cash bond: pay court full amount and get it back when appears Property bond: give court property lien Bonding agent: bond agency has K with court to ensure you show up or they must pay full bond amount o Bounty hunters are non-state actors and not subject to Constitutional limits Dont need warrant, knock and announce, etc. *Some states switched to court-administered bounty programs o Usually underwritten by insurance companies 10% deposit paid to court BRA3142: Preferred form of release is PR or unsecured bond, followed by conditions least restrictive to reasonably assure will appear, including: Custody of designated person who agrees to assume supervision Maintain employment or actively seek Maintain or begin schooling Travel, living or personal association restrictions Avoid contact with victim and potential witnesses Report to pretrial or law enforcement Curfew Refrain from possessing firearm or dangerous weapons Refrain from alcohol or drugs
6

Medical, psychological or drug treatment Agreement to forfeiture property or money reasonably required to ensure appearance o Consequences of Failure to Appear: Bond posted is forfeited Failure to appear can be prosecuted as a separate offense, and then sentenced consecutively with the underlying offense BRA3146(b)(A): penalty for failure to appear depends on the charged crime for misdemeanor, could receive fine or imprisonment up to 1 yr. to be served consecutively o Pretrial DETENTION: should be a last resort BRA3142(e): STD: no condition or combination of will reasonably assure appearance of person and safety of others/community, as shown by clear and convincing evidence Rebuttable presumption of detention IF o current charge is (f)1)[violent crime, life/DPen eligible, drug crime]; AND o has prior (f)(1) conviction within 5 years committed while on release PROOF by: clear and convincing evidence for safety; preponderance for flight BRA3142(i): If judge determines [detention necessary] then he should issue written findings of fact and written statement of reasons Determining Appropriate Release: o Governed primarily by statute: Bail Reform Act of 1984 (or comparable state statute) o First Appearance conducted by Magistrate less formal hearing Hearsay admissible Counsel allowed BUT at this point many s have not been appointed counsel yet Quick average is 2.5min for with counsel; 1.75min for without o BRA3142(f): Detention Hearing: judicial officer shall hold hearing to determine conditions upon s motion when [crime of violence; life/DPen; drug offense OR risk to community]. has right to counsel Opportunity to testify, present witnesses, cross and present information by proffer (we will be able to show) o BRA3142(g) Factors to consider [at first appearance or detention hearing] GOAL: reasonably assure appearance of person AND safety of others and community (1) nature and circumstances of offense: crime of violence, narcotics, etc. (2) weight of evidence [ really has no way to tell this yet] (3) personal history and characteristics including: (A): character, physical/mental condition, family ties, employment, financial resources, community ties, past conduct, drug/alcohol abuse, criminal history, record concerning court appearance AND (B) whether on probation, parole, or other release at time of current offense (4) nature and seriousness of danger to others or community: Can inquiry into lien on property . o *Interlocutory appeal AVAILABLE for detention determinations

CONSTITUTIONAL LIMITS ON BAIL 8A: Excessive bail shall not be required o *NO actual Right to bail, just Constl guarantee that such bail will be excessive *NOT specifically incorporated to states o Excessive: amount higher than reasonably calculated to prevent risk at hand: (1) Primary risk is flight consider: Amount or conditions usually imposed for that offense and circumstances, including typical penalty s evidence that s will flee or pose a risk (2) Secondary risks can be Constl considered so long as detention reasonable in relation to those goals BRA does not violate 8A 5/14A: Substantive DP: o SDP: Govt. cannot engage in conduct that shocks conscience or interferes with fundamental rights
7

STD: does [detention] offend principle of justice so deeply rooted in traditions and conscience of our peoples as to make it fundamental? [offend deep-principle to lockup pre-G] o RULE: Detention Constl IF (1) serves non-penal regulatory purpose AND (2) is not excessive in relation to alternative purpose -- reasonable fit between action and regulatory goal LOOK AT: LegHist statutorily defined class of limited to serious crimes time limits by STA procedural safeguards separate facility (Salerno) 5/14A: Procedural DP o PDP: o RULE: Procedures Constl IF (1) regulatory goal satisfies SDP AND (2) procedures are adequate BRA has adequate safeguards PCause by STD of clear/convincing evidence Counsel right to testify, cross, put on proof, enumerated factors, clear and convincing evidence, judge makes written findings of fact
Stack (1951) EXCESSIVE o [s charged with Smith Act violation crime to overthrow govt. bail uniformly set at 50,000 for each ] o s 8A right to release conditioned upon his adequate assurance he will stand trial and submit to sentence if found guilty o Here, sum much higher than usually imposed for Smith Act and didnt introduce evidence of usual flight risk *bail set at 5 times greater than the maximum fine! Salerno (1987) NOT excessive o facing RICO/conspiracy charges, Mafia ties detained] o BRA is regulatory, not penal o OK under 8A permitting detention for safety risk is not preventative detention or punishment, but Congress solution to societal problem legitimate goal and carefully delineated. Adequate procedural safeguards o OK under SDP detention serves non-penal purpose and is not excessive
o Eg. community safety, quarantine, mentally unstable individuals, enemies during war OK under PDP plenty of procedural safeguards

DISSENT: individuals presumed innocent essentially imprisoning for uncommitted crime w/o proof BRD

IV. CHARGE SCREENING OVERVIEW RULE: NO Constitutional right to charge screening if not taken into custody o GJ not incorporated (Hurtado) does this mean there is a GJ right in Federal System? o DP does not require anything between arrest and trial o 4A only requires detention screening (Gerstein) --WHY BEFORE ARREST? o Most states still have screening process because: (1) permits to mess up and start over w/o DJ concerns AND (2) prevent prosecutorial abuse o RATIONALE: required screening relates to detention under 4As requirement DP does not require state to insure PC before trial GJ right is not incorporated PURPOSE: is to screen the charge and assure PC exists o Also to ensure an unjustly-charged is released from custody or bail conditions AND look for and reduce excessive charges PROCESSES: o Preliminary Hearing o Grand Jury o Direct filing: files information before scheduled PHrg and cuts off screening o Screening upon s motion to dismiss information PHrg v. GJ:
General Feature: Constly required: Burden of Proof: Decided by Remedy for bias? Public hearing? right to be present? right to appointed counsel Hearsay Illegally obtained evidence Cross-exam of witnesses Charge barred by no PC Dismissal for insufficient evid.? RESULT: PHrg NO PC Magistrate YES Yes Yes YES Constlly required feature Allowed; FRE usually n/a Allowed Allowed NO; can start over Probably not Bound over as information GJ Only in Federal system PC GJ YES NO NO NO Allowed Allowed Not allowed NO; can start over NO Issued as indictment

PRELIMINARY HEARING No Constl Right to PHrg DP requires nothing between arrest and trial o Infromation states: ~2/3 states permit prosecution by indictment or information Information must be supported by PHrg bindover, waiver or direct filing via indictment Often only trial gets chance to test the charge and s evidence o Most are very brief (exception child sex offenders) o Discovery conducted by cross-examination has 6A Right to counsel (Coleman) o Considered critical stage of proceeding adversarial NO other Constl rights, just general courtesies o Hearsay admissible o No right to confront witness (Green) o Judge can cut off once PC has been established BoP: Probable Cause: sufficient evidence to support reasonable belief that (1) offense committed (2) by Waivers: o can waive; occurs ~50% counsel practice guides suggest no waiver unless danger to defense Witness available at PHrg but possibly not trial
9

Complainant likely to mellow with time; keep testimony out of record now PHrg accessible to press and could affect fairness of trial later Call s attention to curable defect in his case that he would not otherwise notice until trial Alert to fact that is undercharged o can require PHrg even when tries to waive RATLE: state has independent interest in determining whether sufficient PC exists *see prep for dismissal/bindover consequences

GRAND JURY OVERVIEW o 5A-GJ Right in Federal system for all felonies and capital cases No person shall be held to answer unless on a presentment or indictment of a GJ Presentment: statement issued by GJ without s previous indictment request No Constitl right to GJ in state Ct. not incorporated, but can be in State Const. (Hurtado) o Secret proceeding: FRCrP6(d)(2) Only GJ current witness court reporter interpreter if necessary Neither nor his counsel can be present [But some states permit s counsel] RATIONALE: not only screening but also investigating and deciding who to charge; helps protect witnesses, victims can testify without facing or the general public, prevents s flight since he doesnt know hes being investigated CON: worthless, nearly always assents to , structure problem juries are nonlawyers and take s statements at face value need to curb s power o 16-23 members (federal) and term lasts about 1yr R6(g): can only serve > 18months if court determines extension is in public interest o Selection PROCESS varies between Jxs. Random or Key man: selects jurors with higher education and greater community standing (1) List created using from voter lists, but possibly also drivers licenses, tax rolls,etc. (2) Qualified Wheel: narrows list to those who speak English not blind citizen nonfelon, etc. (3) Array or Venire; smaller group actually selected to sit for GJ (4) GJ chosen from the array in selection process (voir dire for petit) GJ Challenges are either based on either Constitution, statutes or FRCrP 6 DISCRIMINATORY SELECTION o *Discriminatory Challenges are rooted in problems in the qualified wheel or array o Foreperson discrimination: E.g. foreperson, over time, is disproportionately of same [group] Same analysis applies (Rose) *Hobby implied you needed to show that foreperson posses extraordinary power beyond mere managerial duties o STEP1: must show that allegedly excluded group is one subject to oversight by EP clause Race, gender, religion, ethnicity, nationality, age, etc. *could potentially be excluding group that, within specific community, results in discrimination does not have to be a member of allegedly excluded group (Campbell) o STEP2: must make PF showing of (1) substantial disparity in under-representation and (2) underrepresentation originated in part during selection process (1) generally rely on statistical analysis of the GJ array substantial impact: whether exclusion significantly alters GJ composition (McCarthy) absolute disparity: must show >10%difference between groups representation in community and [array] (Mosley) (2) selection based on subjective judgment rather than objective criteria E.g. permitting only Latino workers to be exempt, rather than computer error *dont need to show an intentional discrimination o STEP3: does NOT need to show any actual or likely affect on outcome o STEP4: Burden shifts to to show non-raced based reason
10

o o o

Eg. group excluded not b/c Latino, but agrarian community would suffer injury is its workers were serving jury duty STEP5: Judge decides whether or not race-based RELIEF: Pre-conviction challenge: YES indictment dismissed [w/o prejudice] Post-conviction challenge: YES automatic reversal and indictment dismissed (Vasquez) RATIONALE: EP harms to both and society in general violated either by excluding members of his group OR by violating the rights of the other group excluded from GJ has standing to raise harms of those excluded *post-conviction relief b/c situation not cleansed by BRD conviction
Rose (1979) race-based discrimination necessitated reversal of conviction under EP Vasquez (1986) reaffirmed automatic dismissal as proper relief after post-conviction challenger DISSENT: impossible to tell if race-neutral GJ would not have indictedCt. never suggested that discrimination in composition permits inference that indictments are racially motivated

o o

MISCONDUCT o Constitutional violations: Cant violate or force to relinquish certain Constl rights GJ is Constl required so entitled to have a fair proceeding under 5A Eg. forcing to testify to GJ (threat to lockup if he wont testify) perjured testimony asking GJ to infer guilt from silence BUT many Constl rights do NOT apply here: DJ; 6A-counsel or CC Failure to present exculpatory evidence NOT violation Inflammatory comments courts are mixed, more likely if racially bias o Statutory or rules violation: FRCrP6 procedural errors: Eg. multiple Ws in room while testifying people other than , W, jury present no one during votingdisclosure of GJ matters Statutes: eg. prohibition against using unlawful wiretaps in GJ; crime to suppress perjury o PRE- and POST-conviction challenge: subject to Harmless-Error review TEST: must show (1) violation substantially influenced GJs decision to indictgrave doubt decision to indict was free of substantial influence from violation (Bank Nova Scotia) **TCt not required to rule on the motion pre trial; sine judge can defer until post-conviction, H-E error applies to pre-trial motions as well (Mechanik) o Relief: ONLY if [above test met] then conviction reversed and indictment dismissed *rare to get relief o RATIONALE: Post conviction we have a guilty conviction BRD does it really matter that a technical right was violated early on? Pre-conviction same STD since judge can defer his ruling Misconduct claims will be hard to show dont want to unduly hinder process its supposed to be cursory accusatory not adjudicatory Limit exercise of supervisory power and maintain independence **Misconduct
o o o Mechanik (1986) post-conviction challenge to two witness testifying together; STD is harmless error Subsequent jury decision cleanse earlier error, rendering harmless, unless shows otherwise Bank Nova Scotia (1988) pretrial, argued [various misconduct] no relief unless show error not harmless and had substantial influence Williams (1992): pre-conviction, argued -misconduct for failing to present exculpatory evidence before GJ STD: is harmless error; s chance to disprove and exculpate is at trial *here, no CL ability to quash based on evidence

PERSONAL BIAS o NOT a valid challenge to an indictment or post-indictment conviction (Beck) *no 5A-DP right to impartial GJ lack thereof is not grounds for dismissal o No violation of R6 o could show actual bias on behalf of individual GJ BUT Virtually impossible GJ transcripts difficult to obtain and not likely to reveal bias o Relief: None o RATIONALE:
11

GJ is an accusatory, not adjudicatory body not passing on guilt, only whether he should be brought to trial

EVIDENCE o NOT valid 5A-DP challenge at CL, could not quash indictment based on insufficient evidence keep same GJ function Presentation of hearsay evidence OK (Costello) Weak evidence OK (Holt) No admissible evidence OK (majority) Illegally seized evidence OK (Calandra) o Relief: None o RATIONALE: GJ is method for initiating criminal proceedings and shouldnt be hindered by procedural or evidentiary rules supposed to be abbreviated and cursory dont want to monitor Contrary ruling would permit (and they always would) to request preliminary trial to determine competency and sufficiency of GJ evidence unnecessary waste of time and $

12

V. JOINDER JOINING OFFENSES For each joinder ask: o (1) Can charge together -- Permitted under FRCrP 8(a)? o (2) Can try together-- is joinder so prejudicial that severance should be granted under FRCrP 14? o (3) Does Const require joinder or severance? is essentially required to join offenses if he wants to eventually prosecute all the counts (1) DISCRETION TO JOIN o FRCrP 8(a): MAY charge with 2 or more offenses in one info or indictment IF the offenses: Same or similar character if about the same then join but otherwise unrelated not joinable under here Based on same act or transaction Look at time between offenses usually limited to days or even hours Connected with or constitute part of common scheme or plan Look for a factor connecting the two offenses eg. embezzle to cover up fraud steal car to use after robbery *Applies to felonies and misdemeanors o RATIONALE: PROS: judicial economy; saves $ and time only has to defend once can facilitate concurrent sentences preclusion and DJ issues could prevent mandate joiner CONS: doesnt save *that much time or money (each count has its own Ws and evidence) can confuse jury easier to draw inferences of guilt given risk jury will infer propensity to commit crime thinks with multiple counts is worse offender deserving more punishment problems when wants to present different, inconsistent defenses gets to rehearse **studies show that joinder of offenses increases likelihood will be convicted (2) SEVERANCE May be granted if joinder prejudices o FRCrP 14(a): IF joinder appears to prejudice or , court may order separate trials of counts, sever s or provide other relief (b): before ruling on s motion to sever, court can order to present in camera any of s statements that it intends to introduce at trial. o Spillover Prejudice: argues that evidence from one offense will impermissible show propensity RULE: NO prejudice IF (1) evidence of one offense would be admissible in separate trial fro the other OR (2) if not, evidence of each crime is simple and distinct (Drew) (1) Evidence could be admissible in separate trial if showing: o Motive Intent Absence of mistake common scheme identity of person charge (2) Look at: o Likelihood of jury confusion whether refers to crimes at same time or interchangeable, etc. RATIONALE: jury hears so much evidence that they think must be guilty of something and is a worse offender can also confuse jury
Drew (1964): PREJUDICE charged with robbery and attempted robbery: similarities were same chain of stores, African American in sunglasses but many more differences Here, not similar enough crimes that evidence would be admissible in separate trials Also fails simple and distinct evidence Ws were confused as to which crime referred to; referred to as of the same order; lumped together in closing, etc.

Inconsistent Defenses: wants to testify to one but not the other offenses RULE: Prejudicial if truly wants to remain silent on one offense BUT must (1) proffer proposed testimony AND (2) state believable reason for not testifying on other o Ex parte proceeding needs to be pretty strong reason Court should Balance FACTORS: o Evidence against for each offense
13

Availability of defense evidence other than testimony Plausibility of testimony Effects of demeanor impeachment and cross *courts reluctant to take s word; require a very good reason EXCEPTION: NO prejudice if has a fair choice to take stand or not uninfluenced by consolidation lacked duress (Dunoway) RATIONALE: strong inference of guilt arises when wants to defend one crime but take the 5th on the other BUT easy to assert courts are hesitant to take s word so usually require proffered testimony and reason for motioning for severance
Cross (1964): PREJUDICE charged with robbery of church and robbery of home 2 months later; wanted to testify for one offense Here, seems wouldnt have testified to first count jury wouldnt have heard abt. Prior convicts. or other unsavory activity had no fair choice either testify to both or none

o o o o o

*Drew and Cross test are notoriously difficult to apply before trial hard to tell what sort of effect the evidence will have, whether or not it will confuse jury STD on review is abuse of discretion (3) DOUBLE JEOPARDY - Person can not be put on trial for the same offense o RULE: Same offenses MUST be joined or DJ will bar prosecution of the second offense (1) PRESUME same offense if each does not have a separate element not contained in the other *not same if each offense contains an element not contained in other (Blockburger) Based on statutory elements of offense (2) REBUT by showing Legislative Intent to define and punish as separate offenses E.g.: Armed robbery (Larceny, by force, w/deadly weapon) and Bank Robbery (larceny, by force, from bank) ARE separate . Armed Robbery and Robbery (larceny, by force) are NOT separate Nested offenses: can sent all of them to jury but can only punish for one in offense family [Armed Robbery/Robbery are nested; can send both to jury but only punish one] o CAN motion to sever offenses without a DJ problem is a waivable right o RATIONALE: DJs purposes are to finality, no state of anxiety protect from embarrassment, expense and ordeal of multiple tries prohibit from perfecting testimony from rehearsal, which enhances possibility that innocent will be found guilty [* also same transaction test and Grady test, see class prep]
Dixon (1993) DJ violations for two different s Overruled Grady test and applied Blockburger 1 charged separately for drug possession and criminal contempt DJ barred subsequent prosecution criminal contempt statutes itself imposes no legal obligation swallows drug crime o Contempt: court order + crime on release [cocaine possession] and Cocaine possession contempt has extra element but cocaine does not 2 charged with contempt (extra element is violation of CPO) and threat to kill (extra element is threat)

ISSUE AND CLAIM PRECLUSION Collateral Estoppel *NOT barred by DJ because different victim means its not the same offense RULE: when issue of ultimate fact has once been determined by valid/final judgment that issue can NOT be litigated by same parties in future law suit STD: must prove that jury necessarily determined / relied on disputed fact ASK: can you imagine a reason for acquittal other than rejection of fact now claims was rejected LOOK AT: o jury instructions o proof o record o partys argument o *not likely to look at post-verdict jury explanation
14

*difficult standard to meet (easier in Ashe b/c there was only 1 issue before jury) Special Verdicts: could request one if anticipating long string of prosecutions BUT these are very formulaicgeneral verdicts permit nullification & mercy when guilt otherwise exist RATIONALE: doesnt get two bites at the apple cant rehearse and refine argument; concerned about getting guilty convictions for innocent s
Ashe (1970): [robbery of 6 men, acquitted at first trial with strong evidence of a robbery but weak evidence he was a robber, 2nd trial was found guilty, after refined his case o jry instruction: if was one of participants than theft of any money would sustain conviction, even if he didnt personally take the money here, given instruction, jury verdict necessarily determined that was not present at crime (to find guilty they must find he wasnt a participant cant argue again already bit his apple in the first trial DISSENT: not proper for court to find the reason jury acquitted guesswork/speculation Noth: [charged with M1 based on felony murder statute when murder in course of rape, charge of manslaughter then charged with rape] subsequent rape prosecution NOT barred by collateral estoppel finding manslaughter not murder is not necessarily b/c the rape didnt exists Blockburger presumption subject to idea that LHis can show intent not to treat as same offense

JOINING DEFENDANTS RULE: two or more s may be charged together if they have participated in same act/transaction or same serious of acts o FRCrP 8(b): May charge 2 or more s IF alleged to have participated in same act or transaction OR same series of acts or transaction constituting an offense/offenses. Can be charged in one or more counts together or separately. All s need not be charged in each count. o **MUST be part of same act or series of act if s didnt commit crime together then they cant be charged together NOT sufficient is just same or similar crime RATIONALE: w/o a relationship between s its just two separate sets of culpability do not want jury to assume relationship and use evidence against 1 for 2 RULE: Court may sever s trials, or provide other relief IF joinder appears to prejudice or R14(a) o *hint look for least culpable or facing least serious charges - hell be the one who wants out o Conspiracy: dismissal or failure to prove it does NOT necessarily require severance (Schaffer) must show prejudice from continuing joint trial *Judge has continuing duty to grant severance if prejudice does arise o Spillover and antagonistic defenses: NOT necessarily prejudicial if co-s story undercuts s RULE: no prejudice unless there is serious risk that joint trial will compromise specific trial right of OR prevent jury from making reliable judgment about guilt or innocence (Zafiro) specific trial right: some courts say calling co- as witness is specific right o STD: must show (1) he would call co- at severed trial; (2) co- would in fact testify; and (3) such testimony would be favorable to (Vigil) o Co- Confessions: can NOT introduce co-s confession at joint trial 6A-CC violation (bruton) eg. Cop wants to testify A said me and B did it but B cant necessarily call A to crossexamine b/c A unavailable as witness due to 5A Redaction: removing name from co-s confession is NOT sufficient to remove prejudice violates 6A-CC (Gray) REMEDY: sever trials provide separate juries (expensive, rare) eliminate prejudice and any reference to from confession o CONcern: I did it is just not the same as [deleted] and I did it o *some Cts. require gender-neutral pronoun, others say redaction never ok
15

RATIONALE: limiting instruction in this setting is NOT sufficient. easy for jury to realize deleted name refers to juries cant compartmentalize and imagine confession never heard STD on appeal for misjoinder is harmless-error (Lane)

Schaffer (1960) o [s charged with conspiracy and individually for transporting stolen goods (each made different trip transporting goods conspiracy dismissed] o prejudice is NOT implicit in continuing joint trial after conspiracy charge dismissed BUT judge has continuing duty at all stages grant severance if prejudice does appear Bruton 6A-CC violation to use one s confession in a joint trial, despite limiting jury instruction Gray (1998) o [co- confessed that he, and another beat V to death; confession was redacted by deleting s name] o Bruton rule applies here, where redaction only removes name (eg. me and X) o distinguish Richardson, where the confession didnt suggest s presence, needed linkage from other evidence o REMEDY: several trials or redact so as to remove prejudice by redacting any and all reference to

16

VI. SPEEDY TRIAL AND DELAY INTRODUCTION Constitutional protections always trump statutory o Constl gives the baseline and congres/states can choose to extend protection o *6A reaches some aspects that STA does not: STA doesnt kick in until apprehended Eg. Doggett protected under 6A but not STA b/c delayed in even finding and arresting CONSTITUTIONAL REGULATION *focus here is on time between chargetrial. also DP concerns w/ arrestcharge and trialappeal timing 6A/14A right to speedy trial . different right: o general interest in fairness for AND societal interest in speedy trial
s interests -*impaired defense: -lower chance of acquittal -witnesses die/loose memory -evidence can decay - minimize loss of liberty when incarcerated - minimize anxiety BUT interest in slow trial - more likely to enter favorable plea -postpone going to prison -pretrial publicity and prejudice dies down s interests -risk loosing proof -minimize risk of additional crimes by s on bond BUT interest in slow trial: -convict co-s (no 5A concerns) -secure witnesses -prioritize other trials when full caseload -greater plea bargains leverage Societys interests -minimize risk of additional crimes -quick resolution to crimes shaking community -avoid vigilante justice -costly -decay in witnesses/evidence -Victims rights Act

o delay / deprivation of right can favor o vague when violated depends on circumstances STD: Barker balancing test: o (1) length of delay . triggering factor generally 10months 1yr if less time no violation no right to too-speedy trial; parties need adequate time to prepare *unclear whether prejudice presumed when delay was so long (Doggett yes BUT Reed seems to say no) o (2) s reasons for delay good reasons: getting witnesses, illness of witness bad reasons: anything in bad faith or attempting to hamper neutral factors: negligence, overburdened (slight favor towards ) o (3) s assertion of right *strong factor need extraordinary circumstances when fails to assert still depends on other factors (eg. demand can be outweighed by s good reasons) o (4) prejudice to examine in light of above interests focus on impairing s defense o RATIONALE: specific time period is inflexible and legislative job demand rule bad dont want to infer waiver of Const. right by silence society might want speedy trial when doesnt REMEDY: DISMISSAL o Up to federal judge to decide with or without prejudice o RATIONALE: structural error, harm is not cured by retrial b/c defense is already impaired
Barker (1972) NO 6A violation o [s trial delayed nearly 6yrs, no objection until after 12th continuance; s reasons: procuring co-s conviction to avoid 5A; lead witness ill; caseload full] o didnt assert likely he was baking on co-s acquittal and didnt want first trial o established balancing test REMDY is dismissal no cure given societal concerns
17

o **Other cases are weighed against barker no violation unless situation worse Doggett (1992)YES 6A violation o [ indicted, fled to Columbia, imprisoned in panama. US tried to get him brought back but couldnt finally apprehended in US] o Length8.5 year delay from indictment to arrest o reasons negligence, no serious efforts to locate o assertednot until after arrested but unaware of indictment prior to arrest o prejudice minimal BUT presumed prejudice

SPEEDY TRIAL ACT (STA) enacted in 1974; applies to Federal DCt many states have similar statute Rigidly applied except in extreme circumstances (Eg. Hurricane Katrina) o can NOT waive STAs application (Zedner) STEP(1): Start Clock running [times in days] o arrest 30 days until indictment - 3161(b) o charges filed & made public OR first appearance, later of two 70 days to trial - 3161(c)(1) first appearance before Federal District Judge NO trial less than 30 days [from 1st appear] UNLESS consents in writing - 3161(c)(2) o first day of continuous detainment IF detained under BRA 90 days to trial - 3164 STEP(2): Does clock start over? o clock resets on date that [following orders] are finalized o Dismissal on s motion, then new indictment charging same offense 70 days to trial - 3161(d)(1) o Mistrial or re-trial 70 days to new trial - 3161(e) o Post-appeal retrial 70 days to new trial - 3161(e) retrying DCt. can extend to 180 total days IF witness unavailability/other factors make 70 days impracticable o Post-appeal reinstatement of dismissed indictment 70 days to trial - 3161(d)(2) can extend up to 180 days total STEP(3): Isolate periods of delay o Eg. go through entire process to trial and list out different events and timing between STEP(4): determine excludable delays - 3161(h) o Dismissal of indictment on s motion delay excluded - 3161(h)(6) Contra s motion to dismiss where clock reset here clock just stopped o Pre-trial motion excluded from filing through conclusion of hearing/disposition of - 3161(h)(1)(F) need prompt disposition or no exclusion generally interpreted by Lower Cts. as 45 days to bring motion o Interlocutory appeal excluded - 3161(h)(1)(E) o Trial of other charges against excluded - 3161(h)(1)(D) Eg. charges are severed and its important to finish one trial first, then exclude those periods o Co-s excludable periods of delay excluded - 3161(h)(7) applies when no motion for severance granted o Competency hearing - 3161(h)(1)A) o Delay from transferring case or removing to another district - 3161(h)(1)(G) o Transporting from another district or top/from hospital - 3161(h)(1)(H) presume unreasonably time greater than 10 days from date of order and arrival at destination o Courts consideration of proposed plea agreement - 3161(h)(1)(I) o Delay from absence or unavailability of or essential witness - 3161(h)(3)(A) absent: whereabouts unknown and either attempting to avoid apprehension OR cannot be determined by due diligence unavailable: whereabouts known but presence of trial cant be obtained by due diligence or he resists appearing o *Any continuance granted by judge IF: - 3161(h)(8) (1) advances ends of justice (2) written [or oral] findings in records of reasons why ends of justice served by continuance outweighs s and publics interest in speedy trial *applies to public interests
18

*NO exception to this factor, even if consents (Zedner) FACTORS: failure to grant makes proceedings continuance impossible OR miscarriage of justice so unusual or complex such that unreasonable to expect adequate preparation in time failure to grant in un-complex case would deny reasonable time to obtain counsel or would deny either sides reasonable time necessary to prepare NOT justified by: general docket congestion lack of diligent preparation failure to obtain witnesses on part of STEP5: calculate if over time period then VIOLATION o REMEDY: DISMISSAL if: arrestindictment (30 days) violation - 3162(a)(1) charges filed/first appearancetrial violated - 3161(a)(2) o has burden of proof supporting motion but has burden for evidence relating to [unavailable witnesses] detained under BRA automatic review by court of conditions of release-3164(b) o no detainee shall be held in custody pending trial after 90day period FACTORS for deciding with or without prejudice: o seriousness of offense o facts and circumstances leading to dismissal o impact on re-prosecution on STA and administration of justice o [* will try to argue with prejudice if its or judges fault] SANCTIONS on or s counsel if: o knowingly allows case to be set w/o disclosing witness will be unavailable o files frivolous motion solely for purpose of delay o makes knowing false & material statement for purpose of continuance o otherwise willfully fails to proceed to trial w/o justification Fines: o s appointed counsel reduced compensation o s retained counsel fine not to exceed compensation o Prosecutor fine up to $250 deny right to practice for up to 90 days for either or s counsel file report with disciplinary committee

19

VII. RIGHT TO COUNSEL CONSTITUTIONAL RIGHT ORIGINS: o 6A/14A - incorporated as fundamental right (Gideon) old rule was Betts only required in death penalty and special circumstances o RATIONALE: procedural equality in adversarial system concerned if playing field isnt even we wont ascertain truth bright line rule is easier to administer; hard to predict special circumstances under Betts SCOPE o Crimes: 6A right to counsel applies to: All felony crimes (Gideon) felony: possible imprisonment > 1 year doesnt matter if actual incarceration is imposed or whether ultimately convicted for misdemeanor Actual incarceration imposed for non-felony crimes (Argensinger) 6A requires judge to appoint counsel if he wants to preserve option of sentencing to actual incarceration actual incarceration includes: o conviction and sentence of time served *since its part of the sentence, its considered a penalty right to counsel even though Salerno held pre-trial detention is regulatory, not punitive o suspended sentence (Shelton) *once revocation is triggered, is sentenced for the underlying offense, not the probation violation NOT cured if counsel provided at revocation hearing does NOT include just possibility of incarceration (Scott) o Timing: 6A right to counsel applies for: Critical and adversarial stages of trial: preliminary hearing considered adversarial felony charge plea bargaining trial sentencing FOR NON-felonies, when does it kick in? NOT critical: o pre-indictment line-up o photo identification line-up (post indictment) FRCrP 4(a): right from initial appearance through appeal Direct appeal (Douglas) includes discretionary appeals (Halbert) *basis in 14A EP, NOT 6A no Constl right to appeal but if state provides direct appeal process, then EP requires indigent s have counsel Withdrawal not permitted unless counsel: (Anders) o (1) makes conscientious examination of record o (2) finds it to be wholly frivolous o (3) advises court, requests withdrawal AND (4) provides court with brief referencing anything in the record that could arguably support peal o *Anders is just one method States are free to adopt different procedures for withdrawing counsel (Smith) RATIONALE: poor litigants would be denied EP that rich s had on appeal need counsel to help examine record, research law, form arguments o *at this point reviewing courts looking to merits of claim
20

NO right for post-conviction relief (Ross) BUT state cant adopt practices to entirely cut indigent s off from PCR (eg. high filing fees) RATIONALE: court concerned about extending Douglass holding based on EP/DP any further EP doesnt require absolute equality or precisely same advantages *phases very different at trial attorney is a shield but by appeal he is a sword o by time of appeal, have conviction BRD with counsel o appeal no longer adversarial process

WAIVER RULE: waiver of right to counsel must be intelligent, knowing and voluntary (Tovar) o STD: fully understands nature of right to counsel and its application in general circumstance need not know specific detailed consequences of invoking it o intelligent: knows what he is doing and makes choice with eyes open fact-specific inquiry, LOOK at: s education and sophistication *complex or easily understood nature of charge stage of proceeding (less warning needed for pre-trial) o knowing: judge MUST at least tell : nature of charges right to have counsel range of allowable punishment o *fact based inquiry not per se required to advise that waiving counsel risks overlooking a viable defense loosing opportunity to obtain independent opinion of whether plea is advisable o states are free to impose additional requirements *high standard to meet o RATIONALE: very paternalistic in this setting but not others because: concerned about unreliable judgments and integrity of criminal justice system hindsight s like Tovar who seem confident in decision and later challenge conviction want to make sure theres no way they could later argue it wasnt knowing, voluntary and intelligent
Tovar (2004) [charges were OWI] o judge informed that: entitled to speedy jury trial; right to counsel; if he gives up right to trial, he gives up right to counsel; plea relinquishes right to remain silent, right to presumption of innocence explained maximum penalty for conviction; insured that understood explained elements of the offense o *based on these facts, this and these charges judges admonishments were sufficient BUT really depends on the circumstances

SELF-REPRESENTATION ORIGINS: 6A right o *one of few instances where there is a contrary right also right to testify generally the ability to waive right doesnt give you opposite (waive jury trial doesnt mean right to bench trial) o RATIONALE: *autonomy s right to craft his own defense he must bear consequences of conviction text right to assistance of counsel history no historical norm to representation with attorney BUT: hesitancy viewed as bad choice, self-destructive, inefficient RULE: s exercise of right to pro se MUT be knowing, intelligent and voluntary - unequivocal o STD: whether knew what he was doing and made choice with his eyes open *technical knowledge (FRE) is NOT relevant to assessment of knowing exercise SCOPE: ONLY applies at trial level o right is NOT absolute
21

can be terminated if abuses right/ uses right to disrupt court can appoint STAND-BY COUNSEL preferred practice, but has no right to standby role passive consultant; answers legal questions but NOT there to takeover LIMITS: s right to self-representation is denied if: (McKaskle) o (1) counsel, not maintains actual control of case applies in and out of jurys presence eg. counsel speaks instead of o (2) counsel or judge destroys jury perception that is pro se *objective measure dont look at s subjective opinion RATIONALE: pro se s very sensitive to interference LOOK AT: record, dialogue between D, counsel and judge do NOT actually ask jury can permit hybrid representation (rare) can NOT appeal on grounds of denial of effective assistance or other procedural errors o does NOT extend to appeal (Martinez) RATIONALE: no 6A right to appeal, no historical support *higher govt. interest in accuracy and efficiency REMEDY for denial of right automatic reversal; not harmless error
Faretta (1975) charged with grand theft made unequivocally known he did not want appointed counsel had high school education, represented self before o STD: knowing and intelligently waive right to counsel o LOOK AT: s characteristics here, literate, competent, understanding o technical knowledge is NOT relevant to assessment (cant quiz on rules of evidence) o right to go pro se is NOT a license to abuse dignity of courtroom

INEFFECTIVE ASSISTANCE CLAIMS RAISING: usually NOT raised until after direct appeal process PCR or collateral attacks o RATIONALE: likely same attorney for appeal as trial conflict Direct appeals are limited to record, but on collateral attack can snow pre-trial preparations, communications between and counsel, etc. SCOPE: can only bring claim if proceeding carried a right to counsel o was adversarial, one where 6A would require counsel o same standards apply for appointed and retained counsel Actual denial: o REMEDY: automatic reversal Constructive denial: o STD: attorneys failure is complete (Bell) Sleeping may or may not qualify depends at what part asleep - Burdine (5th Cir) o REMEDY: presume prejudice State INTERFERENCE: or judge interferes with s ability to confer with counsel Eg. no conference for two-day recess IS interference (Geder); 15min recess not o STD: presume prejudice o REMEDY: new proceeding required CONFLICT of Interest: o FRCrP 44(c)(1): joint representation: 2s charged jointly under 8(b) OR s represented by same counsel or associated counsel (2) Ct. must inquire into propriety of joint representation and personally advise each of right to effective assistance, including separate representation take measures to protect s right to counsel unless good cause to believe no conflict likely o STD: shows (1) actual conflict and (2) conflict adversely affected counsels performance (Cuylar) (1) actual conflict: actively pulled in two directions (?) (2) affects performance; does NOT need to show effected outcome eg. changed advice he gave to ; failure to inform o REMEDY: automatic reversal structural error
22

RATIONALE: breach of duty of loyalty; concerned that what becomes good for 1 is bad for the other (eg. 1 accepts plea on condition he will testify against 2)

Strickland ERRORS: o STD: must show reasonably probability that but for (1) counsels unprofessional errors, (2) result of proceeding wouldve been different (1) Error: STD: counsels performance was unreasonable and outside range of professional competence o At minimum: duty of loyalty duty to advocate s cause consult with keep informed *no checklist b/c need flexibility for strategy o LOOK AT; judges should be able to tell; what others do in most cases; can call experts to testify to professl standards Strategic decisions NOT error o *highly deferential review presume reasonable o avoid hindsight judgment. easy to second guess after conviction (2): Changed outcome: STD: reasonable probability that outcome effected o lesser sentence less time, fines, forfeiture, etc. o lesser charge o do NOT need to show acquittal *failure to raise Batson or similar structural claim presume prejudice (most courts) structural: warrants automatic reversal on direct appeal RATIONALE: very difficult to assess whether racial discrimination in jury or similar structural defects effected outcome telling judge about to give perjured testimony NOT prejudicial (Nix) o RATIONALE: provides for more reliable trial not the type of error strickland concerned with constructive denials unless the attorneys failure is complete, NO presumption and must still show [changed outcome] (Bell) o RATIONALE: preserve fairness in adversarial proceeding
Strickland (1984) pled guilty to murder, kidnapping, etc at sentencing hearing counsel presented strategy to minimize cross examination and hope judge would go easy b/c accepted responsibility o decided not to prevent mitigating evidence only interviewed mom, didnt seek character witnesses o Ct. held NOT ineffective acceptable strategy to rely on emotional distress and acceptance instead of mitigation

23

VIII. ACCESS TO EVIDENCE S STATUTORY RIGHT TO S INFORMATION INTRODUCTION o very little discovery right at CL o PROS of s discovery: s statements ensure accuracy discover fabrications if is innocent *encourages settlement if disclosed confession statement, might be more likely to plea o CONS: protect witnesses higher stakes of criminal trial arguably make more likely to intimidate protect ongoing investigations and undercover informants perjury concerns will concoct alibi or defense so as to fit with s case s Oral statements Discoverable - FRCrP 16(a)(1)(A) o upon s request o relevant, before or after arrest o In response to interrogation by person knew was govt. agent o **IF intends to use at trial o RATIONALE: no concern of witness protection; want to ensure accuracy of s own statements s Written or recorded statements Discoverable FRCrP 16(a)(1)(B) o upon s request for inspection, copying or photographing o (i) relevant written or record statement by IF statement in govt.s possession, custody or control; AND knows or should know statement exists o (ii) portion or written record containing substance of relevant oral statement IF made statement in response to interrogation by person knew was govt. agent o (iii) s recorded testimony before GJ related to charged offense o *regardless of whether intends to use s statements to Undercover agents NOT discoverable o doesnt know they are govt. agents o RATIONALE: protect identity and any ongoing investigations s witness lists generally NOT discoverable UNLESS o STD: makes strong showing of special need (Stroop) there, the factors mandating special need were
difficulty to prepare defense (indictment alleges offenses occurring several years ago) complex paper trail multiple s critical, dispositive evidence would flow from anecdotal testimony of other unindicted coconspirators witnesses w/in govt. control and not openly available to had no prior record or involvement in offense involving violence or threat of such s gave no indication that disclosure would increase likelihood witnesses would refuse to testify at trial

o *not specifically mentioned in FRCrP 16 o *some states have discovery provision mandating pretrial disclosure s witness statements Discoverable BUT o must satisfy Jencks Act, codified at FRCrP 26.2 upon s request in possession of govt. relates to subject matter of witness testimony [at trial] statement: defined in 3500(e) and FRCrP 26.2 written statement made and signed by witness, or otherwise adopted substantially verbatim, contemporaneously recorded recital of oral statement contained in recording or transcript of recording statement to GJ, however taken or record, or transcript of such statement *would apply to witness plea agreements o not required to give to until after witness testifies under oath [at proceeding] - 3500(a)
24

Brady NOT violated by Jencks Act IF (1) actually produced after witness testifies AND (2) judge gives necessary leeway to prepare cross BUT in practice, can and often does provide earlier RATIONALE: more efficient; if you wait until after trial testimony then often need continuance so can examine prepare for cross o deceased witnesses: does NOT get their statements Documents and Objects Discoverable FRCrP 16(a)(1)(E) o photo books, papers, documents, data, photographs tangible objects, buildings/places o upon s request o in govt. possession, custody or control AND material to preparing defense, intends to use item in case-in-chief, OR obtained from or belongs to o *does NOT apply/extend to state agency records Govt. reports, memoranda or internal documents NOT discoverable FRCrP16(a)(2) o s work product - opinions about strength of case, whether or not a statement is incriminating, etc. o police reports o RATIONALE: subjecting these to disclosure might encourage statements preparer to be less accurate s Experts reports Discoverable FRCrP 16(a)(1)(F) o upon s request o written summary of expert testimony experts opinion basis and reasons for opinions qualifications supposes that name is disclosed with qualifications o intends to use at trial summary includes: experts opinions Depositions NOT discovery method UNLESS FRCrP 15(a)(1) o shows (1) need to preserve testimony OR (2) exceptional circumstances and interests of justice Eg. witness about to die, being deployed for military service, etc. o RATIONALE: expensive must bear cost for indigent concerned about protection witnesses might potentially have corresponding right to be present at deposition security risks

S STATUTORY RIGHT TO S INCORMATION INTROUDTION o Now discovery is a two way street PRO: evens adversarial playing field already has procedural advantages 5A prevents disclosure of defense, doesnt have to testify, etc. o BUT possible limitations on s discovery: 5A right against self incrimination 6A compulsory process 6A right to counsel Attorney client privilege and work product Reciprocal RULE: DP violation if can discover alibi but has no reciprocal discovery duties (Wardius) o *DP doesnt require states to adopt discovery provisions, but if they do they must be two way s Documents and objects - discoverable IF FRCrP 16(b)(1)(A) o in response to s request for same o [item] is within s possession custody or control o intends to use in his case-in-chief s Reports or Tests o in response to s request to the same o reports of physical or mental examination or scientific test IF o within s possession, custody or control
25

intends to use in case-in-chief OR intends to call witness who prepared report and report relates to testimony s Expert witnesses o upon s request o written summary IF requests same from govt. OR gave notice of intent to present testimony regarding s mental condition o s witness statements no 5A violation: (Noble) choice, not compulsion not s testimony 5A is personal privilege adhering to the person, not incriminating information o Attorney-Client privilege and non-testifying experts: 3 views: does not attach A-C limited to communications btw counsel and attaches IF makes statement to expert/counsel reveals private information attaches whenever attorney or discolses info to expert consulted for purpose of trial prep Notice of alibi provisions: o RULE: must disclose (1) intent to use alibi and (2) names and addresses of witnesses providing alibi defense o Alibi-Notice provisions do NOT violate 5A (Williams) 5A only applies to compelled testimonial conduct that is criminating and personal to doesnt necessarily extent to others statements that happen to incriminate NOT compelled disclosure, just accelerated disclosure still s choice whether to use compulsion would be forcing to testify, etc. o RATIONALE: enhance accuracy and truth-seeking function of trial avoid fabricated, last minute alibi defense *can help by showing he really is innocent s presence at crime is critical element BUT: concerns that leads to new witnesses, unrelated offenses, helps develop rebuttal, tailors case-in-chief (Williams dissent)

REMEDIES AND SANCTIONS - both parties have a continuing duty to disclose - FRCrP 16(c) o evidence/material subject to discovery under R16 AND o other party previously requested or court required its production - If party fails to disclose, court may: - FRCrP 16(d)(2) o (A) order disclosure specifying time, place, manner and other conditions o (B) grant continuance o (C) exclude undisclosed evidence OR *more severe remedy since it effects accuracy of evidence presented to jury see TaylorI o (d) any other order just in circumstances [examples:] instruct jury to assume evidence favorable to other party declare mistrial (rare) dismissal (rare) contempt or fine for violating attorney o **preference for remedy that is least damaging to accuracy of trial - s BREACH o Evidence that contradicts s defense -- FACTORS: culpability negligence or attempt to sandbag prejudice to other pary and whether it can be cured other partys surprise or whether it should have had independent knowledge **importance of evidence to truthful verdict effect on trials accuracy whether it merely impeaches or undercuts foundation of other sides strategy o On review, STD: look at how s breach affected s ability to present defense dont just look at totality and weight of evidence
o People v. Taylor (Mich. 1987) used letter sent to impeach letter flatly contradicted s version of events had duty to disclose but his failure was negligent
26

NO relief exclusion was NOT appropriate remedy: no deliberate attempt to sandbag; wrote ltter and had independent knowledge of it Noe (11th cir. 1987) s version is he was in costa rica present plane ticket, etc introduces [undisclosed tape] which directly undercuts s version *STD: how violation affected s ability to present defense, NOT simply weighing all introduced evidence here, undisclosed evidence attacked very foundation of s strategy REMEDY: new trial

s BREACH o 6A right to compulsory process NOT violated by excluding undisclosed witness- fundamental BUT NO unfettered right to offer incompetent, privileged or otherwise inadmissible testimony MUST follow rules RATIONALE: state interest in orderly conduct of trial interests past just minimize risk that judgment predicated on misleading or fabricated testimony need deterrence o *stricter standard than with s breach for fear that will manipulate discovery rules **shouldnt be punitivem but necessary to insure compliance w/ rules o CON: possibly punishing for his lawyers sins BUT can bring Strickland claim o Taylor v. Ill (1988) provided witness list to but failed to disclose witness who TCt excluded EXCLUSION appropriate lesser remedies not likely adequate deterrents; integrity of entire judicial process at stake

CONSTITUTIONAL REQUIREMENTS Basis: 14A DP clause NOT 6A-CC o RATIONALE: concerned about risk of unreliable factual determinationsjury hearing less than accurate truth False Evidence: o STD: must show knowingly presented false evidence at trial (Napue) o REMEDY: Relief UNLESS shows harmless BRD *typical standard for most COnstl violations except Strickland and Brady Exculpatory Evidence Brady o *claims usually raised at PCR phase failure to disclose usually not discovered until direct appeal o must request the information o STD: must prove: (Brady requirements) Favorable no distinction between exculpatory or impeaching evidence (Bagley) Evidence NO duty to disclose inadmissible evidence (Woods) or s strategy/opinion RATIONALE: cant be material if it couldnt be used at trial Known to or agents working on case NOT extended to independent agency/state In s or agents possession NO duty if knows of but doesnt have access to Eg. witness but identiy not know or hospital record no in possession Material must show reasonable probability of different outcome if it were disclosed reasonable probability: sufficient to undermine confidence in outcome, based on totality of circumstances (Bagley) o BUT look at all circumstances wheter in light of other proof, favorable evidence mightve made difference (fuzzy; hard to do before trial) *more specific requests better more likely that s counsel will rely on and infer non-disclosure means nonexistence o BUT presence of request does NOT itself effect materiality RATIONALE: most debated aspect of Brady; even though difficult for to assess what is material, SCt. doesnt want to force to give up whole case file concerned it would make hesitent to investigate anything that might help Disclosed in time to make fair use at trial [or DPen hearing] if argues too late he must show: reasonable probability that had evidence been disclosed pretrial, result would be different
27

Exculpatory test results or expert analysis most likely to need pre-trial disclosure NOT required to disclose exculpatory evidence before enters into plea (Ruiz) o RATIONALE: s guilty plea means he forgoes fair trial and accompanying Constl guarantees plea OK so long as knowing, voluntary and intelligent Proposed 16(a)(1)(H) would essentially codify Brady upon s request MUST make available all known information that is exculpatory or impeachment can NOT disclose impeachment evidence earlier than 14 days before trial *changes: no materiality requirement
Bagley (1985) D requested evidence relating to deals or inducements pled guilty; discovered via FOIA claims that witnesses had ATF contracts - $300 for info provided evidence disclosed must be material here, remand to assess evidence would have permitted to impeach witnesses; show reward. *s failure to disclose likely to have led to think such evidence did not exist DISSENT: disagrees with materiality requirement should require to disclose ALL favorable evidence s not in position to accurately predict whether or evid. is favorable; depends on s strategy Kyles (1995) reiterates that STD is reasonable probability, less than showing preponderance BUT not really sufficiency of evidence test no need to show that after discounting evidence burden would be insufficient to convict Banks (2004) suppression of evidence in DPel failure to disclose that witness was paid informant MATERIAL testimony indicated CI, not had idea to commit additional crimes given s clean record, testimony was crucial

Witness deportation before able to interview o STD: must show testimony would have been material and favorable (Valenzuela-Bernal) Brady applies o deportation alone NOT sufficient to establish violation o RATIONALE: has manifold responsibilities in immigration cases enforce criminal law; execute Congressional policy of prompt deportation; and look out for human costs to detainee Undercover informants s failure to disclose identity and content of communications o STD: must show identity or communication was material and favorable (Rovario) Brady applies *depends on circumstances, considering: crime chargedpossible s significance of CIs testimony other factors
o Rovario (1957) YES CIs have privilege BUT when identity is helpful & relevant privilege gives way there, CI was sole participant with ; doubts as to whether he recognized YES ID required

Intimidation of s witnesses by or judge o STD: must show testimony would have been material (Webb) o gentle reminding courts divided o advising W or 5A rights less coercive, more acceptable o judge could appoint W counsel and then remind Ws counsel (but rare) o Webb (s only W had long criminal history; judge threatened W with perjury) Refusal to grant immunity to s witnesses who take the 5th o Judge can grant but NO Constl right 6A compulsory process means cant dissuade o STD: (1) whether has deliberate intention to distort [truth]/gain tactical advantage; (2) nonimmunized Ws testimony would be material, exculpatory, and non-cumulative; (3) testimony unobtainable elsewhere (Bahadar, 2d cir) s loss or destruction of evidence o STD: must prove bad faith and evidence could have been favorable (Youngblood) bad faith more than just departure from standard practice or negligence if within routine destruction policy probable NOT bad faith [on behalf of police or ] *Different standard than with Brady where good or bad faith is irrelevant
28

o o
o

Witness IDs: best process is sequential showing; dont say perpetrator is in group; note uncertainties when identified bad showing as whole group, telling perpetrator in there; cross-racial group (YBlood) RATIONALE: limit extent of police obligation confines to class of cases where intests of justice most clearly require it
Youngblood (1988) conviction upheld; failed to meet test police collected clothing but didnt refrigerate shoddy photo ID (above) sex kit analysis tested only to see if sexual conduct occurred; other tests performed 1 month later

REMEDY: if satisfies [above standards] new trial

29

IX. GUILTY PLEAS AND PLEA BARGAINING INTRODUCTION Effective pleas depend on legislature and judges: o ability to bargain depends on where legislatively established sentencing ranges PROS: o for s: efficient saves time and money get cooperation for additional crimes spares victims from trial eliminates uncertainty avoids acquittals on weak case punishment starts sooner increased chance for rehabilitation acceptance of responsibility allows for remorse avoids dangers of pretrial release waives pre-plea errors o for Judges: *consistently impose higher sentences for jury convictions AND accept s recommendation Efficient s express removes waives pre-plea errors makes them just go away o for s **less punishment start sentence sooner allows expression or remorse dont need to risk uncertainty of trial negotiate for other concessions: eg. keep assets get protection dont charge my mother if I plea CONS: o leads to more innocent convicted people plead to get out, need to move on with life even if innocent o privatizes public disputes o many victims want to see perpetrator punished more o arbitrary same conduct can lead to different results disparity Common TYPES: o sentencing recommendation: promises to seek sentence favorable to not seek max; OR refrain from any recommendations BUT still risk that judge will not accept it o lesser included offense generally need courts permission to plead to lesser included o dismissal of [other] charges usually need courts permission FRCrP 48(a): need leave of the court o *other factors promise not to prosecute [family member], etc. Pleas must be voluntary, knowing and intelligent VOLUNTARY voluntary: entered by one fully aware of direct consequences unless coercive induced by threats, misrepresentation, bribe/improper promise o BUT coercion and threat are narrowly defined many threats are hard choices but ok o If elects to sacrifice himself such [improper] motives, that is his choice s threat to charge others NOT coercive IF there is probable cause disparity between potential sentences NOT coercive
30

threatening to seek very high sentence at trial NOT coercive is threat of punishment back legislatively (authority to impose such sentence) threats of additional charges NOT coercive so long as probable cause for additional charges o * not required to charge everything up front if that were the case then he could just use offer to dismiss as leverage s admitted vindictive motive NOT coercive [assuming the threat is non-coercive as well] threat of physical harm COERCIVE lying or active misrepresentation of facts COERCIVE (I think) o BUT puffery is NOT coercive RATIONALE: although hard choices still s choice o s need flexibility in when and what they charge cant be required to charge everything up fron o *Courts and legislatures have not sought to regulate the bargaining process like and need pleas
Bordenkircher (1978) o NOT coercive/involuntary when plea offered 5 years but P threatened to indict under habitual criminal act if didnt accept o *prosecutor admitted vindictiveness BUT very different than unilateral imposition of penalty on for exercising legal right pleas present with hard choices but inevitable part of process Brady (1970) o challenged plea in light of subsequent holding which [held a DPen provision unconstitutional] o not coercive chose to plead guilty aware of precisely what doing o not entitled to withdraw plea once he realizes he miscalculated risks

KNOWING and INTELLIGENT - Intelligent: mentally competent; able to understand attorneys advice - Knowing: MUST: o understand charge and critical elements (Henderson) critical: not obvious & distinguishes charge pled to from lesser offenses (ie M2- intent to kill) either explained by judge at R11 hearing OR sufficient if evidence that counsel explained o understand type and maximum sentence (Williams) maximum term of imprisonment Fines, restitution and forfeiture NOT required to know minimum sentence NOT required to be informed of parole eligibility/ineligibility RATIONALE: considered legislative courtesy BUT most s assume they will get it.. NOT required to know collateral consequences: Recidivist penalties for later crimes Sex offender registry o understand generally waived rights does NOT need specific advisements by judge if explained by attorney does NOT need specific articulation of rights to jury trial; confront accusers; 5A self-incrim o have effective assistance of counsel Strickland STD applies error: unreasonable/ unprofessional o NOT likely errors: misjudging admissibility of evidence (McMann) failing to point out possible defense (Broce) failing to point out potential problem w/ GJ (Tollett) misapprehending strength of s case (Brady) failing to point out collateral consequence prejudice STD: must show reasonable probability that, but for error, would not have pled guilty and opted to go to trial (Hill v. Lockhart) misjudging admissibility of evide o does NOT need to know strength or weakness of s case (Ruiz) o FRCrP 11(b)(1) before accepting, must be under oath and court must inform of and determine understands:
31

s right to use against statement given under oath right to plead not guilty right to jury trial right to counsel waiver of trial rights to confront and cross, protect from self-incrim, present evidence, compel witnesses nature of each charge maximum possible penalty imprisonment, fine, supervised release mandatory minimum forfeiture authority to order restitution obligation to impose special assessment USSG terms that might waive right to appeal or collaterally attack

RATIONALE: judges not particularly paternalistic in this setting; confident adequately counseled

as applied to waiver of counsel generally intelligent is the same general competency stds

BREACH PLEA PROCESS o *breach depends on where in the process we are o Plea K reached by parties federal most are in writing. state jx usually on fly, informal, often oral o Plea and plea K accepted by judge FRCrP 11 hearing o [pre-sentence report prepared] o Sentencing S BREACH o Basis: DP violation o s withdrawal before plea accepted by judge NOT breach UNLESS shows actual reliance detrimental reliance: admission of guilt and confession providing with information testifying for returning stolen goods or restitution failing to file certain motions or waiving procedural guarantees payment of fine or restitution to victim UNLESS govt. can/does repay * could argue withdrawal was in interests of justice BUT many judges not particularly sensitive to s who dont do their homework o s withdrawal for failure to comply after plea accepted but before sentencing BREACH when plea rests in significant degree on s promise such that its consideration (Santobello) must still uphold bargain even when judge admits he wouldnt be affected by it o REMEDIES: to be determined by judge specific performance: resentencing should be with new judge to cleanse poten. error permit s withdrawal and permit trial or new plea *s often like this option, especially if its been several years since offense occurred think s case will have weakened drop or lessen charges FACTORS: degree of reliance whether deliberate or strategic breach stage at which breach occurred s preference should be given considerable weight if possible (Santo. conc.) o *some states let decide o RATIONALE: interests of justice to make uphold his duties; insure gets what is reasonably due
o Santobello (1971) pleads guilty; agrees to make no recommendation; different at sentencing recommends max sentenced to max Need safeguards to insure gets whats reasonably due interests of justice; s need to uphold duties Let state determine appropriate remedy CONCUR: give s preference considerable if not controlling weight Mabry (1984) no detrimental reliance = NO DP violation DPclause NOT a code of ethics for s

S BREACH o s withdrawal after acceptance but before sentencing NO breach if for cause o s withdrawal after sentencing
32

o o

can NOT withdraw unless: (1) plea K has term binding judges discretion in sentencing; (2) judge accepts term; (3) sentenced imposed in that violation (rare) spoiled expectations are NEVER basis for withdrawing plea NO DJ violation if breaches and then re-prosecuted; effectively like DJ waiver (Ricketts) * should be safe and seek term giving him opportunity to litigate whether or not breach occurred cooperation pleas usually: postpone sentencing until after s performance OR promise motion to reduce sentence after performance *permits to hold threat over
Ricketts (1987) plea K- agreed to testify against co in exchange for no life sentence cos conviction overturned refused to testify on retrial plea withdrawn and prosecuted NO DJ violation deliberately chose to terminate proceedings on basis unrelated to factual guilt.. DJ doesnt relive of consequences of his choice

REJECTING DEALS when does judge have to accept and s plea K? *judge CAN accept pleas on condition that they get to review pre-sentence report first coercive CAN reject not intelligent or not knowing (see above) CAN reject disagrees with sentencing bargain CAN reject o [applies when plea k has binding sentence term; if its just recommendation then judge never required to follow] o RATIONALE: sentencing is judges, not s, authority illegal or invalid sentence term CAN reject o Conditions that go to far eg. sterilization, go to church, never ever associate with person X Conceivably violate DP, 1A, EP, etc. o Violation of another statute eg. promising Victim will play no role in proceedings Crime Victims Act 3-4: V has right not to be excluded from public proceedings right to be reasonably heard at public proceedings disagrees with charge bargain can NOT reject (Newman) o EXCEPTION: charge based on: race or gender dislike of s attorney victims wishes (some Cts. BUT VWPA) o RATIONALE: charging decision belong to executive branch disagrees that charges should be dismissed thinks plea too lenient can NOT reject (in re US) o FRCrP 48(a) requires get judicial leave before dismissal BUT judges discretion is to protect and prevent harassment of by repeated filing exchange for dropping civil rights claims can NOT refuse IF has legitimate reason to dismiss charges (Town of Newton v. Rummery) o legitimate could be as simple as protecting victim from testifying at trial waiver of appeal judge generally can NOT reject o waive right to appeal conviction AND [yet-to-be-imposed] sentence AND often 2255 HC review o SoL for crimes can be waived o errors NOT waived: vindictive prosecution DJ situations where constitution forbids from charging at all Can NOT waive errors going towards validity of the waiver (Eg. not knowing, ineffective assistance, coercion, etc) o RATIONALE: parties like because s get finality and better deals; saves appellate cost; leads to stipulations; cleanses errors BUT: unethical for attorneys to waive own errors; skews appellate law development; increases sentencing disparity stipulations can be inaccurate factual basis NOT required under constitution judge CAN accept for crime that did not occur
33

if insists on innocence but wants to plead guilty judge CAN accept if factual basis that (1) offense occurred and (2) committed it (Alford) o *subsequent decision that shows s conduct was not covered by charge and thus innocent: Bowsley STD: can challenge GP on grounds that no longer crime BUT must show also not guilty of any other crimes dismissed as a result of plea K REMEDY if judge rejects on improper ground can file writ of mandamus on interlocutory appeal to get plea entered

34

X. TRIAL BY JURY SCOPE OF RIGHT BASIS: o 6A: right to speedy and public, by impartial jury, in district where crime committed o 14A: (Duncan) o *ArtIII2: trial of all Crimes shall be by Jury WAIVER: CAN waive right to jury trial (Singer) *only waivable Art III mandate o BUT no reciprocal right to bench trial Court can veto s attempted waiver SCOPE: o federal and state crimes o STD: crimes with possible incarceration of greater than six months possible need not be sentenced to actual time (contra right to counsel) must be at least 6 months and a day if at 6 months look at whether additional penalties make it serious (eg. sex registry) o less than 6 months presume NO right BUT rebut by showing additional penalties make it serious o **do NOT aggregate petty offenses 10 petty offenses doesnt get you a jury o fines: can make crime serious corporations statutory minimum is 10,000 BUT generally need **way more** Ct. has found insufficient: 10,000 for union (Muniz) and 3mil. for corporation SIZE: o 12 is normal size no textual reference; determined by history and function protect against tyrannical judiciary community sense of justice; opportunity to participate resist outside influence bribes, corruption ensure accurate fact finding o 6 is Constl minimum (Williams 6 ok; Ballew 5 too small) *post-Williams studies revealed that 6 was worse than 12 for community sense of justice, accurate fact-finding and minority participation UNANIMITY o Federal Ct unanimity required o State Ct NOT required just super majority (Apodaca) 9/12 is the least # so far upheld (Johnson) *no State since Apodaca/Johnson has adopted non-unanimous jury decision NULLIFACTION o jurys power to acquit when evidence and law would otherwise indicate conviction Historical practice, pre-Constitution origins Used in cases when prosecution is unpopular in local community o NOT a jurors right BUT practice is PROTECTED: Ct. can NOT allow directed verdict after acquittal Jury does not need to explain acquittal No special verdicts s attorney can NOT argue nullification or encourage jury to ignore law judge can NOT instruct on right to nullify o LIMITS: cause challenges to jurors Juror cannot serve without taking oath relevancy rules limit evidence RATIONALE: concerned about nullification because: often based on bias, victim stereotypes misunderstanding of law or fact not right way for jury to send a message or seek changes in law want uniform and equal application of laws EVERy elemtn of the crime MUST be found by jury BRD (Gowen)
35

o Gowen applied to 1001 false statements/materiality issue RATIONALE: prevent govt. oppression o safeguards against corrupt or overzealous o concerned about entrusting life and liberty of to one judge or group of judges o Allows for community involvement o BUT: potential for misuse untrained laymen; concerned incapable of adequately understanding evidence; unpredictable

COMPOSITION PROCESS o vicinage: geographic area from which jury pool drawn o Qualified list: federal process in jury selection and service act (JSSA) districts come up with plan in advance - 1863 generally draw from voter registration list, actual voters, or some other supp. sources potential jurors must - 1865 be Citizens understand English be mentally and physically firm not have pending charge or felony conviction [and civil rights not restored] or AG may motion to dismiss indictment or stay proceedings on ground of substantial failure to comply with JSSA BUT such challenge must be brought before voir dire OR w/in 7 days after discovered/could have the grounds therefor- 1867 o venire drawn from qualified list and those listed are summoned to court o voir dire: proceeding to select jurors from venire VENIRE challenges o *6A challenges easier to prove and far more common than EP challenges o 6A Cross-section challenge: STD: (1) must show (1) cognizable group (2) was systematically excluded from qualified list or venire BUT can rebut by showing selection mechanism advances significant state interest (Duren) cognizable group: sufficiently numerous and distinct o sufficient: o distinct: shares specific common charateristics can be unique to community eg. farmers, Indian reservation but NOT youth or certain professions *NOT limited to protected classes systematic exclusion: unfairly represented due to selection system o TESTS: absolute disparity: 10% difference between community proportion and venire proportion *some Jx use comparative disparity: if group makes up less than 10% of population divide absolute disparity by population figure o *not enough to be voluntary exclusion s rebuttal eg. INTERST is caretaker at home; or need to harvest crops has standing even if he is not member of the excluded group (Taylor) RATIONALE: harm is that is deprived of Constlly contemplated jury jury is skewed regardless of whether he is member of group *does NOT apply to actual jury (Holland) REMEDY: new trial (assuming doesnt rebut) Duren (1979) women were 54% of county inhabitants but only 15% on venires. practice where women could decline jury service by responding to selection questionnaire o EP challenge: STD: must show (1) statistical under-representation for suspect class and (2) opportunity to discriminate under-representation: same STDS as cross-section challenges
36

suspect class: gender, race, ethnicity o divided whether religion is suspect class (abt 2/3 states say it is)

JURY challenges o For Cause challenges: *unlimited number of for cause challenges and can raise at any time 6A and DP violation to have bias juror STD: can excluded juror for cause IF has state of mind that will prevent him from acting impartially BUT insufficient if juror declares he can render impartial verdict unless judge thinks otherwise DPen cases cause if juror says he cannot and will not impose DPen or would insist it be applied in every case (Witherspoon) RATIONALE: juror incapable of applying the law implied bias usually limited to relatives or direct employee-employer relationships group association usually insufficient alone Remedy: erroneous grants of cause challenges NO relief EXCEPT for DPen cases o RATIOANLE: no harm; jury still unbiased so got fair trial erroneous denials of cause challenges new trial IF (1) person remains on jury panel and (2) all preemptory challenges used o NO relief simply because party had to waste a preemptory on erroneous denial for cause o Preemptory challenges: practice where both parties have equal number of challenges and can remove juror without cause # of challenges depends on jx and type of case Batson TEST: (1) challenger must establish PF showing that (1) excluded person member of protected class and (2) opponent exercised challenge to remove person on that basis o protected group: race, gender, ethnicity, religion (jx. divided) (2) burden shift to opponent to show group-neutral reason for exercising preemptory o reason can be somewhat subjective or arbitrary so long as believed by judge o LOOK at: whether other jurors to who reason applies were similarly discharged (Miller L) eg. argues women discharged b/c unemployed were other jurors similarly unemployed (3) judge decides if challenger proved by preponderance purposeful discrimination Remedy: usually new jury after trial new trial after jury selected but before trial dismiss jury and start over (same or new venire) o *most common before full jury selected reinstate improperly dismissed o *if not possible, then do NOT get to replace dismissed with person of same group (point is to not base any juror selections on group membership) attorney SANCTIONS not generally imposed but could be for repeated Batson violations RATIONALE: enhances s acceptance of result; lets him walk away feeling like jury was fair decreases appellate ligitigation dont need to appeal denial of for cause challenges subjective and powerful BUT available to both sides BUT preemptory challenges in general are contested feature of CJ system some suggest each party could have 1 allow it as back up for cause o Voir Dire questions: Generally very little needs to be asked; judge retains broad discretion in limiting questions 6A right to ask racial prejudice questions when: DPen-eligible case with interracial murder (Turner) significant likelihood that racial prejudice might infect trial 14A-DP (Ham)
37

o *very narrow application; Ham arose in highly contested civil rights era For non-DPen racial-related cases, supervisory power requires judge to permit such questions but NOT Constl right (Ristaino)

TRIAL RIGHTS Right to be PRESENT o right applies at proceedings that satisfy Stincer TEST: (1) 6A-confrontation violation IF s exclusion interferes with effective cross-examination of witness at trial (2) 14A-DP violation IF s presence would have (1) helped defend self at trial OR (2) resulted in more reliable outcome LOOK for: factual finding that had effect on admissibility of evidence; some issue could weigh in on
Stincer (1987) pre-trial hearing in chambers to determine competency of child victims; not present but counsel was questions only dealt with competency, not substance no 6A violation subject to full and complete cross, present during cross and could help counsel questions asked in competency hearing could be repeated at trial no 14A violation questions not based on substance no indication that had special relationship or knowledge that could assistance counsel/judge in asking more accurate questions

plea negotiations no 6A problem; just two lawyers bench conference regarding witness competency no 6A problem Forfeiture due to s DISRUPTIVE BEHAVIOR: STD: can loose right to be present IF (1) after having been warned by judge that he will be removed if continuing disruptive behavior (2) nonetheless insists on conducting self in manner so disorderly, disruptive and disrespectful that trial cannot be carried on with him in court room (Allen) judge MUST inform he has right to come back when he decides to behave self suggested methods for unruly : cite for contempt: o CON not likely to be effective contempt and discontinuance until orderly o CON: some s might intentionally misbehave to delay trial (try to lose W) confine in restraints o BUT can NOT be prejudicial: (Deck) applies to trial and sentencing belly chain and shackles not ok invisible restraints possible stun belt controversial; argues so scary cant concentrate RATIONALE: undermines presumption of innocence Remedy for improper removal: new trial NOT harmless error Forfeiture due to NON-APPEARANCE: at start of trial: generally MUST be present (Crosby) BUT some states waive if (1) knew trial start date & was (2) voluntarily absent shows up and then fails to return for remainder (Taylor)
FRCrP 43:(a) s presence required at: initial appearance, initial arraignment and plea every trial stage including jury impanelment and plea sentencing (b) NOT required in following circumstances: organization misdemeanor offense conference or hearing based on question of law sentencing correction (c)(1) waiver: initially present at trial or pled guilty waives right to be present when o voluntarily absent after trial begun o noncapital case where is voluntarily absent during sentencing
38

court warns he will be removed for disruptive behavior and he persists in being disruptive (2)trial may complete through verdict and sentencing if waives right

Right to FAIR trial s IMPROPER ARGUMENTS NOT permitted o Basis: DP, not supervisory power personal opinion on Ws credibility or s guilt Ct.s vary: very sweet witness advocacy BUT rape V as fine girl improper issues beyond s guilt or innocence play on fear, subsequent conduct of , unnamed inflammatory passion or prejudice attacking integrity of s counsel *misstating law *reference facts outside record *reference to s silence and its implication *racist remarks o STD: whether s comments so infected trial with unfairness as to make resulting conviction denial of DP (Darden) NO DP violation if simply undesirably or universally condemned LOOK AT: type of violation (eg misstating law worse than impassioned statement) isolated, brief episode or continuous argument whether invited response to s argument strong evidence of guilt (s comments would make less difference) whether objected (might not object in strategy to try to get overruled) whether jury giving curing instruction [*heightened fairness needed in DPen cases (Darden dissent)] o Remedy: new trial BUT NO reversal unless above satisfied (Modica) PREFER: interrupt counsel forcefully instruct jury grant mistrial reprimand infront of jury place in contempt refer to disciplinary proceedings suspend Appellate court can: o reprimand by publishing s name in opinion o initiate disciplinary hearings o counsels improper arguments: generally held to same requirements some Cts. think more appropriate to play on emotion *rarely deal with this issue only when analyzing invited response so egregious as to require mistrial and challenges under DJ TJudge cuts off argument and challenges as reversible error ALSO: o compulsory process o Public trial o Confrontation clause o Right to testify o Right to remain silence

39

XI. SENTENCING INTRODUCTION TRENDS: o Increased rate of incarceration from 1920s onward war on drugs movement from parole and towards determinate sentencing determinate: fixed term set by judge w/o early release on parole, but followed by period of supervised release indeterminate: judge sentences to range and leaves to parole board to decide, based on rehabilitation when gets out o increased number of African-Americans incarcerated compared to other groups o increased proportion of spending on prison systems in state spending truth in sentencing policies: giving states more federal funding if they abolish parole for more violent offenses competing GOALS: o equality controlling disparity in sentences; wanted similarly situated s committing same crime to get similar penalties o individualization preserving discretion so as to let punishment fit individual, not general offense SENTENCING DISCRETION Prosecutors: o charging decision gives initial role in deciding sentence Judges: o Statutory Minimums: either minimum sentence for certain offense OR mandatory minimum when judge finds existence of certain facts o Guidelines: USSG now advisory (1) start with base offense level (2) add specific offense characteristics eg. harm caused, role in offense (3) downward departure for acceptance of responsibility o codified plea discount (4) assess criminal history category based on individual s record (5) look at chart where offense level and criminal history category intersect is your range CONSTITUTIONAL LIMITS PROCEDURE: o after GP or sentencing, probation officer prepares pre-sentence report s RIGHTS: o right to be present o right to public proceeding o right to counsel o judge required to allow to allocate BUT Cts divided whether this is a Constl right *usually just right guaranteed by statute o NO right to jury UNLESS sentencing fact is equivalent to element [of greater offense] DPen can be imposed by judge if sentenced authorized after capital conviction and no further facts need to be found in order to impose o NO right to confrontation o NO right to discovery or pre-sentence disclsoure EXCEPT DPen Surprises ok, secrets not judge MUST disclose portion of PSR upon which he relied, but right is limited to time of sentencing (Gardner) Federal felony, usually gets PSR (FRCrP 32(e)(2) at least 35 days before sentencing) o NO right to compulsory process or right to present evidence
40

JUDGE CAN RELY ON pretty much ANYTHING no limitation shall be placed on information concerning background, character and conduct of person convicted of offense which [fed court] may receive and consider for purpose of imposing appropriate sentence - 3661 o Sources outside record and hearsay includes Pre-Sentence Report (PSR) (Williams v. NY) NO DP violation RATIONALE: historically judge had wide discretion in determining sentencing practical reasons evidence rules designed to save time and prevent confusing jury with collateral issues keep trial limited to guilt v. innocence *policy seeking individualized sentence means need more information in line with goals of reformation and rehabilitation o Other crimes and non-charged criminal conduct judge must make finding that conduct occurred by preponderance of evidence applies if was charged and acquitted acquittal is failure to meet BRD, can still meet preponderance RATIONALE: goes towards likelihood of rehabilitation individualization permits judge to determine where within range this should fall o s commission of perjury (Dunnigan) BUT just MUST review evidence and independent findings so as to establish willful perjury, not just confusion, mistake or faulty memory preponderance STD RATIONALE: goes towards s ability/respect for law, general character o Cooperation and failure to cooperate Cooperation makes sense to consider to help reduce sentence Failure to cooperate *much harder call concerned about s 5A silence right CAN rely on non-cooperation like refusing to name drug supplier BUT judge can NOT use s refusal to cooperate / provide information as a direct inference that s proof was correct; find fact based on s 5A right (Mitchell) o Elements of character such as Racism IF relevant to prove aggravating circumstance White-supremacist convict of crime lacking any indication of racial motivation NOT ok to use (Dawson) RATIONALE: 1A violation proves nothing more than abstract beliefs Anti-semite convicted for arson of Jewish center OK as sentencing factor (Kapadia) RATIONALE: relevant to dangerousness and possibility of rehabilitation o Victim Impact Statement YES can be used, even in DPen cases (Payne) Victim has right to appear and be heard at sentencing RATIONALE: harm is relevant to degree of punishment even though not really elated to s blameworthiness

APPRENDI ISSUES Apprendi Line Statutory maximums o Williams judicial fact-finding to set sentence within range; no increase max penalty ceiling o Alemendarez-Torres sentencing enhancements based on prior convictions is Constl judge must determine prior conviction by preponderance NOT and element of the offense dont need to allege in indictment RATIONALE: history judges have generally relied on priors to raise sentencing; fed ct. and all states had such statute on books *problematic case; many judges now say they disagree o McMillan/ Harris judicial fact-finding to set higher minimum sentence is Constl (Eg. robbery is 5-10 years; with gun minimum of 7) NOT an element, just sentencing fact o Apprendi has 6A right to proof BRD by jury for facts raising statutory maximum (Eg. robbery is 5-10; but with gun can raise to 12 years)
41

*viewed as an element of a greater offense like the aggravated offense Apprendi satisfied if admits aggravating fact or such fact established by conviction
Apprendi (2000) [ pled to possession of firearm for unlawful purpose; statute permitted judge to raise max (10yr) if crime was racially motivated (meant judge could go >10) Ct. held s 6A rights applied these are the sorts of facts we want juries determining not as objective and clear cut as prior conviction

Ring (2002) Apprendi applies to aggravating factors in DPen sentencing EXCEPTION: DPen can be imposed by judge if sentenced authorized after capital conviction and no further facts need to be found in order to impose Booker line Sentencing guidelines o Blakely (2004) has 6A right to proof BRD by jury for facts raising mandatory SG maximum (Eg. statute says 10yrs max; mand. SG calculated at 5 yrs max unless judge finds aggravating facts, then increase to point above 5 [but still below statutory max]) extended Apprendi to Washington State SG mandatory guidelines: if judge departs from SG range then parties can appeal (downward- can; upward can) Blakely (judge raised for deliberate cruelty SCT rev o Booker (2005) Blakely extended to USSG [which were then mandatory] Part 1 H: violates 6A jury trial right for judge to find aggravating fact not established by the conviction Part 2 H: strikes provision making USSG mandatory now USSG are advisory STD: on review is reasonableness for departures from USSG recommendations
[Booker: 10-life(stat); 17y6m-21y10m (USSG) judge found additional drugs and obstruction, sentenced to 30y Fanfan: max was 78m right after Blakely but judge found additional facts but said he couldnt follow USSG]

Post Booker world: o USSG are advisory makes it like Williams STD: preponderance is sufficient *judge of course must still stay within statutory max or else Apprendi violation o ASK: is this a Williams fact or an Apprendi fact? o Alternative sentencing schemes: single, mandatory sentence CON: no individualization indeterminate parole sentences use juries to determine aggravating facts for mandatory SG CON: consumes resources inverted guidelines: Legislature sets very high maximums and then proves mitigating factors topless guidelines: only minimum; skys the limit for max o RATIONALE: USSGs goal is to tailor sentencing to real conduct, not just conduct proven for conviction o Problems: Almendarez-Torres criticized that its still good law post-Apprendi Should Restitution and Forfeiture fall under Apprendi line? restitution: $ to be paid back to victim as part of sentence, determined by amount of loss NOT statutorily set fine forfeiture: requires judicial finding at sentencing that property was instrumentality or from proceeds of crime o viewed like Williams but not authorized unless judge makes factual findings

42

XII. DOUBLE JEOPARDY INTRODUCTION Basis: 5A: nor shall any person be subject for the same offense to be twice put in jeopardy of life or limb o 14A applies to states (Benton) GOALS: o FINALITY allow and V to know this is the end o Accuracy more likely to convict innocent s if has chance to rehearse and try again o Proportionate punishment dont let get more than one bite at the apple and extract more punishment by going after again and again does NOT operate between sovereigns: o Multiple states can prosecute same conduct INITIAL INQUIRIES: Has DJ ATTACHED? o jury trial jury sworn in (Crist) RATIONALE: s interest in retaining chosen jury and having them complete the process o bench trial first evidence presented: first witness sworn OR first document admitted o guilty plea plea accepted by judge o RATIONALE: until these points, s interest in finality, accuracy or punishment arent that strong are there any issues of Same offense? o RULE: Same offenses MUST be joined or DJ will bar prosecution of the second offense (1) PRESUME same offense if each does not have a separate element not contained in the other *not same if each offense contains an element not contained in other (Blockburger) Based on statutory elements of offense (2) REBUT by showing Legislative Intent to define and punish as separate offenses E.g.: Armed robbery (Larceny, by force, w/deadly weapon) and Bank Robbery (larceny, by force, from bank) ARE separate . Armed Robbery and Robbery (larceny, by force) are NOT separate Nested offenses: can sent all of them to jury but can only punish for one in offense family [Armed Robbery/Robbery are nested; can send both to jury but only punish one] MISTRIAL REPROSECUTION IF DJ attached sought to stop trial: o STD: NO DJ bar UNLESS goads into mistrial leaves w/ no choice but to motion (Kennedy) *difficult to apply; LOOK for: whether s action was deliberate whether objected to s motion (shows he had wanted continue) how well trial is going for whether he had motive to goad s merely harassing or overreaching conduct sufficient for mistrial BUT NOT DJ bar o *some states reject Kennedy and bar retrial when s motive is merly harassment or inject prejudice o RATIONALE: if s the one who wants to stop then he shouldnt mind starting over did NOT seek to stop trial: o STD: DJ BARS re-prosecution UNLESS manifest necessity to grant mistrial manifest necessity: impartial verdict cannot be reached or reversal is certain (Somerville) Jury bribed information/indictment fails to state a crime deadlocked jury (Perez) o LOOK AT: whether objected length and complexity of trial length of deliberations
43

potential impact of exhaustion or coercion whether forcing more time would be coercing just a few jurors to change minds o RATIONALE: not acquittal or conviction, but ending trial b/c cant reach impartial verdict NOT manifest necessity if judge could cure problem substitute juror or proceed with 11 give instruction grant continuance make jury deliberate longer makes one or two outbursts fails to line up his case (key evidence not ready) o *could be strategic move tor try to get retrial if NOT manifest DJ prevents retrial b/c first trial ended unnecessarily

Kennedy (1982) o charged with theft. during trial s attorney impeached expert by mentioning he filed complaint against ; on rehab said is it because [ is a crook]? o motion for mistrial granted SCT held NO DJ bar elected to start over o here, wanted to continue; shocked at mistrial isolated incident early

DISMISSAL or ACQUITTAL REPROSEUCTION Pre-Trial dismissal NO DJ bar (Serfass) o RATIONALE: DJ hadnt attached Dismissal following jurys guilty verdict NO DJ bar (Wilson) o RATIONALE: not put in jeopardy twice if successful appeals the dismissal then verdict simply reinstated Dismissal after DJ attaches on grounds other than sufficiency of evidence or s guilt/innoc. NO DJ bar o insufficiency of charge or defective indictment (Lee) o pre-indictment delay (Scott) o Statute of Limitations o Improper venue (Wilkett) o can generally appeal and if dismissal reversed can re-prosecute o RATIONALE: rulings that do not touch on sufficiency of evidence (not equivalent to acquittal) *many states require such motions be made pre-trial and are barred/waived once jury sworn Dismissal on grounds of insanity or entrapment DJ BARS re-prosecution (Moore) o RATIONALE: defenses that bear on culpability make him less guilty or justified Dismissal on grounds of recantation [in false testimony before GJ hearing] NO DJ bar (Moore; DC Cir) o RATIONALE: defense not bearing on guilt or innocence Dismissal based on legal ruling on sufficiency of evidence DJ BARS re-prosecution (Sanbrias) o fails to meet burden of proof. evidence excluded (that renders proof insufficient) etc. *judge can NOT grant acquittal before rests his case o DJ still bars prosecution when judges grounds for granting motion of acquittal were erroneous error to exclude evidence (Sanbrias) error to grant b/c judge though s misconduct and witnesses soo bad/incredible (Fong Foo) o Sanbrias motion granted for judgment of acquittal on finding that s evidence to one theory was inadmissible and therefore insufficient o RATIONALE: viewed essentially as equivalent of acquittal Acquittal after trial DJ BARS re-prosecution o EXCEPTION: acquittal procured by bribery or similar fraud Implied acquittal rule DJ BARS re-prosecution of greater included offense (Green) o jury asked to determine guilt on greater and lesser included offense and jury returns verdict convicting of lesser offense but remains silent on greater offense imply that jury acquitted of greater charge o rules does NOT apply when: hung jury jury notes it cannot reach agreement on higher charge
44

acceptance of guilty plea ( never really been in jeopardy of the higher charge) *of course would be barred in that must abide by his part of plea K

POST-CONVICTION REPROSEUCTION Appellate relief/reversal after convicted appeals procedural errors NO DJ bar (Ball) o applies to faulty jury instructions relevancy of evidence Blakely errors o RATIONALE: waives finality of verdict when he appeals choosing to set aside finality *policy-based no such thing as a perfect trial if retrial was barred after every appellate finding of error too many people would go free Appellate court finds TJudge shouldve granted judgment of acquittal / finds insufficiency of evidence DJ BARs retrial (Burks) o RATIONALE: regardless of who determines it, insufficiency of evidence ALWAYS is DJ BAR

45

XIII. REVIEW OF ERROR BY APPEAL OVERVIEW NO Constl right to appeal o All states current have right to appeal, although for GPs its often just discretionary review IF State provides right to appeal then Constitution REQUIRES: o NO vindictive prosecution in response to exercising right Pearce; DP if gets relief on appeal, then cannot charge higher and judge cannot sentence higher o Must provide counsel for indigents on direct appeal Douglas; EP o Must provide TCt transcripts for indigents on direct appeal Griffin; EP s APPEALS STEP1: ID specific error and STD of review to see if can even meet the showing o Abuse of discretion: applies to decisions requiring judges extra info acquired from presiding over trial Eg: Evidentiary rulings Sanctions for discovery abuse Challenges for Cause *hard STD for to show o Clearly erroneous: applies to findings of fact, Eg; whether juror intended to use preemptory based on race for Batson challenge whether client was advised on maximum sentence before pleading o De Novo: applies to mixed questions of law and fact Strickland 1 whether counsel was reasonable dangerousness at bail hearing manifest necessity for mistrial, etc. no deference to TCt STEP2: Can appeal at this time? o Final judgment rule: can NOT appeal until sentence imposed; EXCEPTION RATIONALE: more efficient to resolve all errors at once appeals can create undue delay and lead to lose of evidence/less accurate outcome BUT: could prevent costs of needless trial, doesnt force to disclose trial strategy o *s like b/c H-E error is not available o Collateral Order RULE: can seek interlocutory appeal for orders so unrelated to ultimate decision of proof of elements IF: (1) order conclusively determines question Ruling can NOT be tentative, informal or incompleteeg. defer, or reasonable prosecpt that Tct might change ruling in light of facts (2) issue is important and independent of merits wouldnt be affected by subsequent decision on merits (3) Normal appellate review would provide inadequate remedy review after final judgment would not cure error; harm would have already occurred CAN seek interlocutory appeal for: Denial of bail Certain conditions of pretrial release (requiring to go t church (interferes w/14A) Double Jeopardy or Speech Debate clause Motion for return of property (nothing to do with proof of guilt) Contempt order against witness Forcing to be medicated for competency (bodily harm would occur by sentence) can NOT seek interlocutory appeal for: vindictive prosecution (but some Cts. split) Motion to dismiss due to speedy trial violations Grand jury errors
46

Improper joinder (fails #3; could just redo with separate trial) Denying s demand for discovery Order requiring produce discovery o UNLESS s counsel refuses and is placed in contempt STEP3: Did expressly waive right to seek relief on error o MOST pre-plea errors are waived EXCEPT cannot waive errors affecting validity of waiver itself (coercion, ineffective counsel, etc) o stipulations o on-record waivers o jurisdictional waivers (?) STEP4: Did forfeit right to seek relief by pleading guilty? o MOST pre-plea errors are cleansed when plea accepted EXCEPT errors in plea-taking process (ineffective counsel) errors that cant be cured vindictive charging GJ selection challenges racial discrimination errors expressly reserved in conditional plea (eg. denial of motion to suppress) STEP5: Did forfeit right to seek relief by failing to raise error in timely and appropriate manner? o RULE: must raise or its waived. NO relief UNLESS PLAIN ERROR FRCrP 52(b) STD: must show (1) error [steps1-3 above] (2) plain: obvious clear error or problem o something arguable would not be clear problem (3) substantial effect on outcome o seriously affects fairness and integrity of proceedings o Eg. need additional facts such as [error] affected ability to put on defense failure to prove element of crime would affect outcome (BUT will argue it could have been cured had raised it) o * 52(b) is permissive not going to correct things that dont need it RATIONALE: provides safety valve some issues so fundamental that CJ systems integrity would be undercut if they went uncorrected high degree of importance failure to raise may not be s fault concealed evidence, law since changed o RATIONALE: want parties to raise errors so TJudges can fix them when they occur more efficient prevents sandbagging getting error in record without mentioning until appeal challenge incentives to follow rule by properly raising errors need deterrence STEP6: Assuming error not waived or forfeited RELIEF? o Relief required regardless of harmlessness: Particular statute mandates relief Prejudice or materiality already established as part of error Strickland claims *DP violations where must show effect in order to establish claim itself o Bagley evidence and family o Youngblood o Darden (improper argument) o doesnt live up to bargain in plea o misrepresentations in plea negotiations by o improper to grant severance under R14 Bars prosecution and punishment entirely Double Jeopardy Speedy Trial Violations Vindictive prosecutions
47

**require dismissal Structural Errors: aka Automatic reversal Right to effective counsel Right to impartial judge Right to public trial Failure to properly instruct that burden of proof is BRD Discrimination in GJ and petite jury Right to self-representation Race-based use of preemptory Improper exclusion of for cause juror in DPen case [no longer coerced confession (Fulminante overruled Payne)] RATIONALEs: o inability to measure effect of error so pervasive throughout process that we couldnt predict what outcome would have been like [w/o error] o interests protected are those OTHER than verdicts reliability discrimination in jurors, personal autonomy, etc Other errors NO relief if error is HARMLESS: *applies to most properly preserved errors Constl errors STD: must show BRD that error did not contribute to outcome (Chapman) show rational jury would have found guilty even w/o error Subject to H-E standard: Misjoinder of s Brady evidence failure to turn over Coerced confession Failure to tell max fine at plea hearing Failure to turn over R16 evidence (not Brady evidence) Improper admission of co-s statement Improper R8 joinder Denial or right ot counsel at PHear Failure to establish factual basis at Plea hearing Denial of cross-exam at detention hearing Omitting element from jury (Neder) Commenting on s silence (Griffin) Denial of allocution at sentencing (Cts. are divided)
Chapman (1967) [s convicted of robbery, murder, kidnapping, got death/life] jury instructed it could draw inferences from s failure to testify after trial, Griffin held such instruction was UnConstl (1) federal law applies to errors of state law dealing with Constl issues (2) CAN have harmless Constl errors here, NOT harmless errors repeatedly impressed upon jury that inference from silence should be drawn in favor of state lots of circumstantial evidence REMEDY: new trial

S APPEALS Controlled by statutes can appeal any order so long as no DJ problem - 3731 (p.239) can appeal pre-trial suppression/exclusion of evidence IF USAtty certifies that: - 3731 o appeal not taken for purpose of delay; AND o evidence is a substantial proof of fact material in proceeding can appeal sentencing decisions [Other statutes may also grant right to appeal] STANDARDS OF REVIEW * Sentencing departure reasonableness (Booker)
48

XIV. POST-CONVICTION: HABEAS CORPUS INTRODUCTION Direct appeal: trialappellate State SCt US SCt. o If SCt doesnt grant relief or denies cert . can file motion for PCR in State trial court o *usually first place to raise ineffective assistance of counsel claim trial appellate State SCt US SCt if still no relief after US SCt then can file writ of habeas corpus for review in FDCt 2241: applies to persons objecting to detentions w/o conviction military custody; pre-trial detention, etc. 2244: applies to all persons in custody pursuant to judgment of Federal Ct. 2254: applies to State prisoners in custody o in custody: incarcerated OR parole / probation incarcerated at time petition filed challenged to conviction/sentence to be served after current civil commitment or contempt o RATIONALE: gives federal court chance to correct unlawful detention and serious errors *really only worst felony s are still in custody by time they get to this point in process BUT need federal comity towards state authority AEDPA (1996): o bound Fed Cts. to time period to review DPen cases certain # days once you start in process BUT contingent upon State providing counsel to all Death row prisoners on STate PCR proceedings no state has yet been able to qualify for it o imposed SoL for filing have within 1 yr from time direact appeal is finalized to file Federl HC *SoL tolled while State PCR proceedings occur o authorized to deny on merits any claim raised for first time in Fed HC if it had not been exhausted through state courts (before, could only dismiss w/o prejudice and let try in state PCR) o STD of review: relief only when state has unreasonable application of Federal law STATE PROCEDURAL DEFAULTS *situations where failed to properly present claim in State court in accordance with applicable state procedural requirements IF fails to comply with state procedural rule requiring to raise claim [in certain manner and at certain time] and SCt refuses to consider claim on direct appeal and raises in Fed HC writ o STD: claim BARRED unless shows (1) Cause and Prejudice OR (2) Miscarriage of Justice Cause and Prejudice: must prove cause for default o good reason failed to raise o *usually established through ineffective assistance of counsel *default must occur at state where had right to counsel Coleman NO cause for failure to raise during state PCR must prove prejudice o actual difference in outcome Miscarriage of Justice aka actual innocence exception STD: more likely than not that NO reasonable juror would have convicted House first and only case to satisfy so far o *showing of probable innocence is NOT itself Constl claim; must still show some constitutional violation o *satisfying above tests only means that Federal court will now review the claim o RATIONALE: deference to state court; prevent sandbagging STD of review for properly raised or preserved claims Harmless error o Structural errors properly raised still automatic reversal o BUT structural errors NOT preserved plain error on direct appeal and must satisfy procedural default for Fed HC HYPO: H-E Never raised in State TCt plain error STd on direct appeal procedural default STd for Fed HC
49

Das könnte Ihnen auch gefallen